CFP

¡Supera tus tareas y exámenes ahora con Quizwiz!

What is the present value of an investment that has the following inflows/outflows if the client's required rate of return is 10%? End of year 1 pays dividend of $100 End of year 2 pays dividend of $150 End of year 3 no dividend paid, client invest $1,000 End of year 4 no dividend paid End of year 5 investment sold for $18,000

$10,640.15 CF CLR WORK-> 0 Enter v (CF0 is always 0)->100 Enter vv->150 Enter vv->-1,000 Enter vv->0 Enter vv->18,000 Enter vv->NPV->10 Enter v-> CPT

A client wants to **receive** the equivalent of $10,000 in after-tax dollars at the beginning of each year for the next 4 years. If the client can earn 8% after tax on their investments, how much must they invest today to fund this need if inflation is 5%?

$38,363.98 1.08/1.05-1X100=I/Y->10,000 PMT->4 N-> CPT PV

Bond Conversion Value

(Par / Conversion Price) x Stock Price

How to calculate FICA Tax

(Social security tax rate X Gross Pay) + (Medicare tax rate X Gross pay) = 6.2% (up to $147,000 of Gross Pay) of Gross Pay + 1.45% of Gross Pay

Self Employment Tax Rate

.1413 (14.13%)

Is this life insurance included in estate? 1 You own the policy and die 2 You own the policy on your spouse and you die 3 You gifted your spouse's policy to your child 4 Spouse is owner of policy, you gifted it to them and you died within 3 years of transfer 5 Spouse is owner of policy, you gifted to them and never changed the beneficiary 6 You sold your policy

1 Death benefit included in gross estate 2 Replacement cost is included in your probate estate 3 Nothing included in your estate 4 Death benefit included in your gross estate (3 year rule) 5 Death benefit included in your probate estate 6 Nothing included in your estate

Deductible or Not Deductible: 1 Investment interest expense 2 Car loan interest 3 Credit card interest 4 Mortgage interest 5 Student loan interest

1 Deductible (up to the amount of net investment income) 2 Not Deductible 3 Not Deductible 4 Deductible (for either 1st or 2nd mortgages) 5 Deductible

Examples of: 1 Risk Avoidance 2 Risk Diversification 3 Risk Reduction 4 Risk Retention 5 Risk Transfer

1 Don't buy, rent; don't buy a house with a pool. 2 Spread assets/activities across different areas/markets/products 3 Sprinkler system/smoke detectors; home security system 4 Deductibles in insurance policies; coinsurance in insurance policies 5 Purchase insurance; hedge

1 Jensen(alpha)/Treynor 2 Sharpe

1 Measured in terms of *BETA*, volatility, contains systematic risk Look for high R2 (60+) or diversified portfolio Look for highest positive alpha, then highest Treynor number 2 Measured in terms of *STANDARD DEVIATION*, variability, contains systematic AND unsystematic risk Look for low R2 (less than 60) or non-diversified portfolio, look for highest Sharpe number

Probate or Non-Probate: 1 JTWROS/Entirety 2 Estate is the beneficiary 3 Tenancy in common 4 General Powers 5 Gift Taxes paid - 3 yrs 6 Singly owned assets 7 Community Property 8 Life Insurance

1 Non-probate 2 Probate 3 Probate 4 Non-probate 5 Non-probate 6 Probate 7 Probate 8 Non-probate

Transfer for value exceptions

1 Transfer to the insured on the policy 2 Transfer to partner of the insured on the policy by the insured 3 Policy transfer to partnership where insured is a partner 4 Policy transfers to a corporation where insured is officer/shareholder

CASE Harry invested $10,000 on January 2nd (year 1) in a balanced mutual fund. He held the fund and then sold it on January 2nd (year 4) for $15,000. The fund declared the following dividends and capital gains for each year: - Year 1: Dividends $100 (reinvested) STCG $100 (reinvested) LTCG $150 (reinvested) - Year 2:Dividends $50 (paid in cash) STCG $100 (paid in cash) LTCG $100 (paid in cash) - Year 3:Dividends $150 (reinvested) STCG $150 (reinvested) LTCG $150 (reinvested) Question 1 What is the cost basis at the time of sale in year 4? A. $10,800; B. $11,100; C. $10,550; D. $3,900; E. $10,250 Question 2 What is the internal rate of return over the holding period if the dividends and capital gains are paid at year-end (December 31st)? A. 9.44%; B. 12.16%; C. 12.89%; D. 14.25%; E. 15.2% Question 3 What amount of dividends and capital gains have to be reported as taxable income in year 3? A. $150; B. $300; C. $450; D. $4,200 HINT: Count the sales proceeds as part of the 3rd year cash flow

1. A. $10,800 Original $10,000 + Year 1 $350 + Year 2 $0 + Year 3 $450 = $10,800 2. E. 15.2% CF->CLR->-10,000 Enter v->0 Enter vv->250 Enter vv->15,000 Enter v->IRR CPT 3. C. $450 The taxable income will be $150 of dividends, $150 of STCG, and $150 of LTCG from year 3.

CASE Hal and Patricia have been seeing each other for 18 months. They cannot decide whether or not to get married. To try things out, Hal is giving up his apartment and moving in with Patricia. Patricia owns her home outright. She also has a brokerage account, a 401(k), and a medical plan at work. Question 1 When Hal moves into Patricia's house, will Hal's personal property be covered? A. Yes, they have a common-law relationship. B. Yes, Patricia gave Hal permission to move in with her. C. No, Hal needs a renter's policy. D. No, Hal is like "a man without a country." Question 2 If Patricia re-titles her home in JTWROS with Hal, is there a taxable event? A. No, Hal does not have an economic benefit. B. No, not until the home is sold. C. Yes. When Patricia adds Hal to the deed, this constitutes a taxable event and counts against her lifetime gift exemption (less the annual exclusion). D. Yes, but it is taxed as a life estate (a gift of a future interest). Question 3 What would happen if Hal died prematurely right after the home was re-titled? A. Patricia would get a full step-up in basis. B. The IRS will try to include the whole value of the home in Hal's estate. C. Hal's ownership will pass to his family members. D. The home will be subject to probate. E. The IRS will not recognize JTWROS between an unmarried couple. Question 4 If Patricia adds Hal to her brokerage account, is there a taxable event? A. No, Hal does not have constructive receipt. B. No, not until Hal withdraws funds from the account C. Yes. When Patricia adds Hal to the account, this constitutes a taxable event and counts against her lifetime gift exemption (less the annual exclusion). D. Yes. However, it is taxed as a remainder interest (a gift of a future interest). Question 5 Hal and Patricia have no legal documents in place. Which of the following legal documents should not be considered as a minimum to protect their assets? A. Domestic partner agreement for dissolution of their relationship B. QTIP trust C. Revocable trust D. Durable power of attorney Question 6 If after living together two years they decide the house is not big enough and sell it, how much 121 exemption can they claim? A. None, they have not lived there five years. B. None, they are not married. C. $250,000, since she has lived there more than two years. D. $500,000, since they have both lived there more than two years. Question 7 What should a CFP® professional recommend Hal and Patricia do? A. They should get married since there could be several tax benefits and they could save more money with their reduced living expenses. B. If they have been dating this long and still cannot decide if they want to be together, they should probably call it off. After moving in and commingling assets and property, the break up could get expensive. C. Hal should buy a vacation home. D. They should see a relationship counselor to see why they cannot fully commit and keep all their assets separate for now.

1. C. No, Hal needs a renter's policy. 2. C It says she re-titled the home. He has rights under the deed. This is a gift. 3. B The IRS will assume that the first to die owns the entire property 4. B. No 5. B. QTIP trust They are not married, so they can't fund a QTIP 6. C. $250,000, since she has lived there more than two years. 7. C Answers A, B, and D all have the CFP giving relationship advice. That is not the CFP® professionals job. Any answer, no matter how off-the-wall, that contains no relationship advice would be correct.

Community or Not Community Property: 1. Property received as gift/inheritance that is used to buy an asset in one spouse's name. 2. Property received as gift that gets commingled with community assets. 3. Assets obtained by spouses prior to marriage. 4. Property received as gift or inheritance by one spouse.

1. Not community property 2. Community property 3. Not community property 4. Not community property

Practice Standards Steps (1-7)

1. Understand client's personal/financial circumstances (qualitative and quantitative info) 2. Identify and select goals 3. Analyze current course and potential alternatives 4. Develop recommendations 5. Present recommendations 6. Implement recommendations 7. Monitor

A CFP® professional must: 1. Act with _____, _____, _____, and _____. 2. Act in _____. 3. Exercise _____. 4. Avoid or disclose and manage ____. 5. Maintain the _____ and protect the _____ of _____. 6. Act in a manner that _____.

1. honesty, integrity, competence, and diligence. 2. the client's best interests 3. due care 4. conflicts of interest 5. confidentiality, privacy, client information 6. reflects positively on the financial planning profession and CFP® certification

How is American Opportunity Credit Calculated

100% of first $2,000, then 25% of next $2,000 Max of $2,500

What rate are collectibles taxed at?

28%

If investor can earn 8% on his investment and inflation is 4%, what is the investor's real rate of return?

3.846% [(1.08 / 1.04)-1]X100 = 3.846%

Calculate AIME

35 highest earnings years / 420 = AIME

Are the premiums paid for group term life insurance a deductible business expense by the company? A. Yes B. No C. Only if the plan does not discriminate in favor of key employees D. Only if the employer is indirectly a beneficiary under the policy

A Answer C is incorrect. The plan costs are deductible even if the plan discriminates in favor of key employees. The key employees will be taxed on the first $50,000 of benefits. Answer D is incorrect. The plan is only for the employee's benefit, not the employer.

Andy wants to accumulate $150,000 for retirement in 8 years. He expects to earn an annual return of 13% compounded semiannually on investments. How much should Andy invest at the beginning of each 6-month period to attain his goal? A. $5,264.45 B. $5,606.64

A. $5,264.45 *Beginning* of each period, so BGN mode. Clr work->BGN->P/Y=2->150,000 FV->13 I/Y->8xP/Y, N->CPT PMT = $5,264.45

When can a CFP® certificant reveal confidential information? I. To comply with legal process II. In connection with a civil dispute between a CFP® certificant and a client III. To defend the CFP® certificant against charges of wrongdoing IV. Even if it causes irreparable harm to the client V. With the client's consent A. All of the above B. I, II, V C. I, III, V D. I, V

A. All of the above

Which of the following will not go through probate: A. JTWROS B. Revocable living trust C. Tenant in common D. Tenancy by entirety E. Testamentary trust

A. JTWROS, B. Revocable living trust, D. Tenancy by entirety

72(t)

Allows penalty-free withdrawals from IRA accounts and other tax-advantaged retirement accounts Owner of the retirement account must take at least five substantially equal periodic payments

Sharpe (Hint: standard)

Almost identical to treynor, except use standard deviation instead of beta Portfolio return - Risk free rate / SD R2 below 60

Bond Current Yield formula

Annual Coupon / Face Value

Trina Sawyer has the following income for this year. What amount of self-employment tax must she pay? - Sole proprietorship net income$65,000 - Reimbursed entertainment expenses$ 7,000 - Subsidized parking for the year$ 3,000 - Wages from an S corporation$25,000 A.$4,945.50 B.$9,184.50 C.$10,597.50 D.$12,717.00

B $65,000 x .1413 = $9,184.50 Reimbursed entertainment expenses, subsidized parking, and wages are not self-employment income.

Alan Newsome has a net worth of $300,000. He is married with two children. His investment advisor is suggesting a private placement issue. He is only suggesting using 20% of his net worth. Alan's only investments are in low risk type CDs, mutual funds, and ETFs. Alan has worked hard, sometimes two jobs since high school to accumulate his net worth. What is Alan's situation? A. Dire B. He is considered a non-accredited investor C. He is considered an accredited investor D. As a low risk investor the broker/dealer will not allow the purchase

B He does not have the net worth nor income (not shown) to be an accredited investor. Dire is not an answer. This is a Regulation D question. With regards to Answer D, not enough information is known about the private placement to make that judgment.

Mr. Todd, age 63, currently owns a $1,000,000 life insurance policy on his life. The beneficiary was his wife. Mrs. Todd died a year ago. Mr. Todd just changed the beneficiary to his revocable trust. Which of the following is true? A. He violated the transfer for value rule. The policy will be income taxable. B. It will be included in his estate should he die. C. It was included as an asset in her estate when she died. D. It will be included in his estate under the three-year rule for another two years.

B He is the owner/insured. He can change the beneficiary anytime. By naming his revocable (no irrevocable) trust as the beneficiary, it is included in his estate. This does not violate transfer-for-value. His wife, at death, was only the beneficiary of his policy. The three-year gift rule does not apply here. It would only apply if he tried to give his policy to someone else, like his daughter or his irrevocable trust.

What is the most important need? A. Life insurance B. Health insurance C. Umbrella liability insurance D. Emergency fund

B Health insurance needs to be a priority and poses the greatest risk. Without knowing if they have kids or not, it is more difficult to prioritize the life insurance, but it is more important than umbrella insurance. The primary goal of an emergency fund is to provide stability in case of a job loss. Think about the possible size of the risk. How much does three months without a job cost versus three months in the hospital?

What are differences between TIPS and I-bonds? A. They are both government securities that are indexed to inflation B. TIPS purchases are unlimited per year; I bonds purchased by an individual in a given year is limited to $10,000 C. TIPS trade like ordinary Treasuries; Saving bonds cannot be sold until maturity D. TIPS income is taxed as ordinary income; I bond taxation occurs when the bonds reach maturity or are redeemed

B I bonds can be sold after one year (Answer C). Answer A is not a difference. Answer D should say interest income. Phantom income is charged the appreciation on face value of TIPS. It is not collected until the TIPS are sold or it mature.

On July 30th, an XYZ DEC 75 call has a premium of $8, and XYZ shares have a market price of $78. Which of the following are true? I. The call has no intrinsic value. II. The call has an intrinsic value of $3. III. The call has an intrinsic value of $5. IV. The call has a time value of $3. V. The call has a time value of $5. A.I, V B.II, V C.III, IV

B IV = MP (78) - EP (75) = 3 points in the money; therefore, it has an intrinsic value of $3. The rest of the premium represents the time premium.

BGN or END: Retirement benefits/income

BGN

What is covered by Medicare Part A and Part B? A. Custodial care B. Dentures C. Yearly wellness visit D. Hearing aids

C Be careful since custodial care is another way of describing long-term care. Answer C is available once every 12 months. It is not a physical exam. Answers B and D are not covered.

Maggie gives $5,000 in cash to her church this year and this is her only itemized deduction for the year. Her AGI is $50,000. Which of the following is false? A. She should take the standard deduction. B. She can take the full $5,000 as an itemized deduction on the Schedule A. C. She can take $300 as a deduction for AGI. D. She will be limited to 60% of her AGI on cash gifts to charity.

C Due to the consolidated Appropriations Act of 2021, non-itemizers could have taken up to $300 single, $600 MFj as an above the line deduction in 2021. This is expired. Answers A, B and D are true. She should take the standard deduction and not itemize.

Which of the following can be deducted on the 1040 this year? A. Gambling winnings B. Alimony received C. IRA contributions D. Fees to financial advisors

C Winnings and alimony received are income. Fees to financial advisors are no longer deductible.

What is FICA taxable? How much is taxed?

Compensation 7.65% (6.2% Social Security up to $147k, 1.45% Medicare)

Medicare Part B

Covers doctor services, diagnostic tests, blood transfusions, radiology, physical/occ. therapy

Medicare Part D

Covers prescription drugs

BGN or END: Mortgage payments

END

Risk Diversification

Exposures are spread across a variety of areas, markets, or products Same as diversifying a portfolio

T/F: Assets with a named beneficiary are subject to probate.

False

T/F You can't have both Medicare and employer coverage at the same time.

False Group health plan pays first, and Medicare pays second

Defined Benefit Pensions (favors, benefit, cash, credits)

Favors older employees Certain retirement benefit Must have very stable cash flow Past service credits allowed

Simple IRA (employer type, match, limit, plans)

For small employers (100 or less employees) Requires employer match (immediate vesting) Salary reduction limit up to $14,000 Company cannot have another plan

GST Tax applies to

GST tax applies to transfers to related individuals who are more than one generation away — such as grandchildren or great-grandchildren — and unrelated individuals, like family friends, who are more than 37½ years younger

2503(c)

Hold gifts for child until they reach age 21

Put Option Intrinsic Value

IV = EP - MP

Holding Period Return Formula

Income + Ending Value - Beginning Value / Beginning Value

Schedule B

Interest and Dividend Income

Money Purchase Pension Plan (deduction, cash, contributions)

Max contribution is lesser of 100% of salary or $61,000 Up to 25% employer deduction Employer wants to retain key employees Wants plan that is simple to administer Employees are young and well paid Must have stable cash flow to make fixed contributions

Risk Reduction

Reduce financial losses by implementing measures that reduce the impacts of potential losses

Duration is an important measure of bond risk because it measures _____ to _____ changes.

Sensitivity, interest rate

Three-Year Rule How does is apply to gifts

When individual transfers an asset within three years of date of death, value included in gross estate Gifts fmv amount above the annual exclusion and taxes paid on the gift is included in estate

Schedule A

itemized deductions

Sally has asked CFP® practitioner to evaluate the following bonds. She is in a 37% marginal tax bracket. On a tax-equivalent basis, which of the bonds provides the best return? NOTE: Unless it says the investment is subject to the 3.8% Medicare tax, do not use the 3.8% in any calculation. A. Municipal bond paying 5.0% B. Corporate zero bond paying 7.8% C. Treasury zero bond paying 7.6% D. Corporate bond paying 7.9%

A The zero's interest is taxable each year. It is not deferred. The tax-equivalent yield of the municipal bond is 5.0% : 63% = 7.9365%. This way you only have to do one calculation and can compare all four bonds (apples-to-apples). Treasury bonds, even zero coupon bonds, are subject to federal tax, but not state or local tax like a step-up.

If a retiring client expects to live on 50% of their current pre-retirement income adjusted for inflation, what do they need to consider least? A. Their income taxes B. Their investment performance C. Their risk tolerance D. Their life expectancy

A They will probably be in a lower tax bracket than when they were working, but the question says 50% of their income. The amount of income they actually receive will be based on investment performance and risk tolerance. Receiving income is likely more important than paying taxes. Life expectancy is also important.

In which situation is the supply of shares limited? A. Closed-end fund B. Open-end fund C. UIT D. No-load balanced mutual fund E. Answers B and D

A UITs issue units, not shares

Which of the following gifts would constitute a taxable gift? A. $32,000 to your son B. $20,000 to a college to cover your friend's tuition C. $16,000 to your granddaughter D. $32,000 to the Republican party

A Unless it indicates it is a split gift, a gift of $32,000 to your son is a taxable gift. The question is written in the singular. A gift directly to a college for tuition is tax free gift. The gift to the granddaughter is a direct gift. Gifts to political parties are exempt. There is no annual exclusion.

ABC owns a large citrus grove. ABC is concerned about an oversupply of South American juice being imported into the U.S. Which of the following hedge positions should ABC take? A. A short hedge - ABC should sell OJ futures contracts because it is hedging against lower OJ prices B. A short hedge - ABC should sell OJ futures contracts because it is hedging against higher OJ prices C. A short hedge - ABC should buy OJ futures contracts because it is hedging against lower OJ prices D. A long hedge - ABC should buy OJ futures contracts because it is hedging against lower OJ prices

A When ABC owns the OJ, it is long in OJ. (However, it does not yet have the orange juice. It will be long when the oranges are harvested and juice squeezed.) ABC would hedge against lower orange juice prices in the future by going short (selling) a contract of orange juice at today's prices for delivery tomorrow (when selling prices may be lower). This would hedge against the South American juice reducing OJ prices. In Answer C, they are buying contracts. This means owning again. Owning does not offset owning.

The annual returns for XYZ for the past three years were 8%, 12%, and -6%. What is XYZ's geometric return? A. 4.12% B. 4.37% C. 4.67% D. 8.64%

B (1.08 x 1.12 x .94)= 1.137 CLR->1.137 FV->-1 PV->3n->CPT I/Y = 4.37%

After doing a lot of research on a mutual fund, a CFP® professional is ready to present her analysis. The mutual fund is highly correlated to the market (R is 90%). The fund has a realized return of 8% with a standard deviation of 12%. The return on the market is 8% with a risk-free rate of 4% with a standard deviation of 10%. Which of the following statements should the CFP® professional make? A. The fund should be purchased because the Sharpe ratio is high. B. The fund should not be purchase the alpha is negative. C. The fund should be purchased because the Treynor ratio is high. D. With a realized return of only 8% the standard deviation will cause the fund to underperform correlated funds.

B *With a high R2 you should always select the alpha answer.* You cannot use Sharpe with high a R2. Treynor by itself is meaningless. Even though the alpha is only slightly negative this is probably a large cap fund. Other large cap funds will probably have positive alpha numbers. First you have to solve for beta. R is .9. R is the correlation coefficient. You have to square it. It is still above .6 or 60%. Beta = correlation (mutual fund SD) / market SD = .9(12%)/10%=1.08 α = Rp - [Rf + (Rm - Rf) β] = 8% - [4% + (8% - 4%)1.08] = 8% - [4% + 4.32] = 8% - 8.32 = -.32%

When will the proceeds from a life insurance policy not be subject to income tax? A. When a client sells his policy to his ex-spouse B. When a client gifts her policy to her daughter C. When a client sells his policy to a viatical company D. When a client buys a policy from her corporation

B Answer A is a Transfer for value, the policy is being sold and there is no mention of a divorce decree. Answer C is a Transfer of value as well, going to a viatical company. Answer D is too vague, buying "A" policy from her company. The insured can buy their own policy, but not someone else's. This is not clear as to what is happening. Answer B is the best answer here.

Mrs. Perry has an AGI of $145,000. She recently donated $100,000 of publicly held stock to a private university. The stock was purchased 10 years ago for $25,000. What is the maximum allowable income tax deduction she can take in the current year on this gift? A. $25,000 B. $43,500 C. $50,000 D. $100,000 E. $145,000

B Long-term appreciated property (FMV) is allowed a deduction of 30% of her AGI

Mrs. Lucy, age 80, is in reasonable health. Five years ago, her husband died leaving her $5 million and placing $5 million in a bypass trust for her benefit. In addition, their home was in JTWROS. The home has an FMV of $1,500,000 and the $10,000,000 of investments has a high basis of $8,000,000. Mrs. Lucy has two married children and 5 grandchildren and wants to keep her estate under $11,000,000. What type of asset should a CFP® professional recommend she give and to whom? A. Low basis, high dividend paying investments to both children and grandchildren. B. High basis, high dividend paying investments to both children and grandchildren. C. Low basis, growth investments to children and high basis, growth investments to grandchildren. D. High basis, growth investments to children and low basis, growth investments to grandchildren.

B Think about their ages. The grandchildren have to be age 30 maybe age 40. With high basis, they can sell the investment with little or no tax or keep it and get big dividends. They could be taxed a 0% or at most 15%. Low basis investments would be subject to capital gains. If she keeps the low basis stocks until death, her estate will get a full step-up in basis. She does not have an estate tax situation. There is no one right answer. All you can do is to select the one that accomplishes the best purpose of the gift.

A client buys two puts. October ABC put @$50, ABC selling @ $52 January XYZ put @ $30, XYZ selling @ $26 What is the intrinsic value of the two options? A. $-2/$4 B. $0/$4 C. $0/$-4 D. $2/$-4

B Think simple — You cannot have a negative intrinsic value. Only one answer has no negative.

Alvin is employed by ABC, Inc. The company provides a group disability (salary continuation) plan that pays Alvin $5,000 per month if he is disabled. The company pays 50% of the premium, and Alvin pays the other 50% after-tax (salary deduction). If Alvin is disabled, are the benefits taxable? A. The $5,000 benefit is tax-free. B. $2,500 is tax-free, and $2,500 is taxable. C. The $5,000 benefit is fully taxable. D. Split premium plans are no longer available

B This is a contributory plan (50%) by the employee. Half of the benefits are tax-free and the other half taxable.

Tilly put $100,000 in a trust for her grandson. During his high school years (4), college (4), and graduate school (2), he will receive all the income from the trust. After completion of the 10 years, the remaining trust assets will be returned to Tilly. Taxation of the yearly income will be the responsibility of whom? A. Tilly as this is a remainderman interest B. Tilly as this is a reversionary interest C. The grandson as he is a remainderman D. The grandson as he received the income

B This is a reversionary interest. The trust assets revert to Tilly. It is currently tainted for income tax purposes. It will also be tainted for estate tax purposes if the reversionary interest exceeds 5% of the trust's corpus of death.

In general, bonds... I. pay interest at the beginning of the period. II. pay interest at the end of the period. III. are issued at par value. IV. pay semiannual interest. A. I, III B. I, IV C. II, III, IV D. III, IV

C If you also considered zeros as part of the answer, then there is no answer. All the answers were wrong.

Which of the following trusts does not qualify for the marital deduction? A. QDOT/QDT B. QTIP C. Dynasty (B trust) D. Marital A

C The QDOT/QDT is like a QTIP, which does qualify for the marital deduction. A dynasty trust is for children and grandchildren. It qualifies for the lifetime exemption.

Mrs. Baker, single age 75, wants to contribute $70,000 of cash to State College. Her AGI this year is $100,000. How much can she deduct this year if she itemizes? A.$30,000 B.$50,000 C.$60,000 D.$70,000

C The SECURE Act established a 60% AGI cap on gifts of cash to charity.

A fire destroys ABC Inc.'s building this year. The insurance recovery ABC receives results in a gain. What tax options does ABC have? A. It has to report the gain. B. It can build a new building from the insurance proceeds. C. It can acquire qualifying replacement property within two years to defer all or part of the gain (involuntary conversions) D. None, casualty losses cannot be deducted.

C We tried to give you a hint with involuntary conversion. Under Section 1033, an involuntary conversion is defined as a destruction or loss of the property through casualty, theft, or condemnation action pursuant to government powers of eminent domain, and the resulting compensation from such destruction or condemnation. Even though the sale and/or compensation for the property were essentially forced on the taxpayer, the taxpayer is still liable for any capital gain tax liability on the compensation received. However, if the property is subject to an involuntary conversion, the taxpayer has the ability to defer the payment of the depreciation recapture and capital gain taxes on the involuntary conversion under the non-recognition provisions of Section 1033. NOTE: 1033 still applies for real property exchanges.

Iris works for XYZ, Inc. She is a participant in her company's 401(k) plan. She defers $1,000 at the end of each month, and the company does a 50% match. She already has $10,000 in the 401(k). If she makes 6%, how much will she have in the 401(k) in 20 years if none of the facts change? A. $497,453 B. $495,143 C. $726,163 D. $659,959

C Yes, the payment must go in as a negative. This is cash flow out. If you inputted the payment as a positive, you got Answer D. Pretty hard to retire on a negative FV. CLR->12 P/Y->-10,000 PV->-1,500 PMT->6 I/Y->20xP/Y N->CPT FV

Mr. Williams' yearly compensation is $80,000. He contributes $10,000 to his 401(k). Mrs. Williams does not work outside the home. They have no other taxable income. How much can they contribute to an IRA and deduct in 2022? A. $6,000 B. $7,000 C. $12,000 D. $13,000 E. $14,000

C You can only assume that he is under the $109,000 (given in test material) active participation phase out because no other taxable income numbers are given. Therefore, he can do a deductible IRA. She can contribute to a spousal IRA ($6,000) because he has compensation. She is under the spousal $204,000 phaseout. You cannot assume they are age 50.

If the beta of a stock is 1.2, the risk free rate is 5%, and the return on the market is 8% then, according to the CAPM, what is the required return for the stock? A. 3.6% B. 6% C. 8.6% D. 9.6% E. 13%

C r = rf + (rm - rf)β = 5% + (8% - 5%)1.2 = 5% + 3.6% = 8.6%

Toby and Mary Welch, ages 67 and 66, have retired. They are leaving for France for a 3 week vacation. What type of insurance will cover emergency care while in France? A. Medicare A and B B. Medicare B only C. Medigap Plan F D. Medicaid

C Medicare coverage will not cover outside the U.S. except for Canada, Mexico, the Carribean and cruise ship excursions. Answer C, Medigap, is the only possible answer. Another answer is Travel Medical Insurance but that was not one of the options.

Sara and Jon Billings bought a house 10 years ago. They financed $200,000 with a 30-year 5% mortgage. If they sell today, how much mortgage principal will remain? A. About $134,000 (2/3 of $200,000) B. $175,282 C. $162,684 D. $159,132

C. $162,684 CLR TVM->P/Y=12->200,000 PV->30xP/Y N->5 I/Y->CPT PMT = $1,073.43->2nd AMORT->v 10xP/Y Enter->v Balance will show($162,684.13) ->v principal paid will show->v interest paid will show

Due to very extreme inflation, the yield curve is inverted. Short term bonds are yielding 20% and long term bonds only 14%. The government and the Fed intend to reduce inflation to under 10% by trying to move the curve from inverted to normal. What should a CFP® professional suggest their client buy assuming all the following bonds have the same quality? A. 2-year bond B. 7-year bond C. 12-year bond D. 20-year bond

D You want to lock in that 14% return for as long as possible. This is covered in the Live Review Investment section. The Fed is going to attack the 20% and reduce it. Last time this happened was 1981. The inverted curve will change to a normal curve and the client can lock in a high long-term yield for many years.

A 13-unit apartment project costs $700,000. -It has 13 two-bedroom apartments renting for $750 per month. -Laundry income is $1,000 per year. -Vacancy and collection losses are 7% of potential gross income. -Operating expenses are $44,250 for this year. Calculate the yearly net operating income (NOI). A.($19,592) B.$43,672 C.$64,560 D.$65,490 E.$65,560 With a 12% cap rate, what is the maximum price you would advise the client to pay? A. $545,750 B. $914,500 C. $975,000 D. $983,333

D; A 13x750=9,750x12 = 117,000 + 1,000(other income) = 118,000(PGI) - 8,260(vacancy and collection) = 109,740 - 44,250(op expense) = $65,490 Intrinsic Value = NOI/CapRate = 65,490/.12 = 545,750

401(k)

Deferral limit of $20,500 Employer wants qualified plan, but can only afford minimal expense beyond salary and benefit costs

Medicare Part C

Health insurance plan that provides Medicare benefits through private insurance company

During the year, Fred Smith had the following expenditures for his rental house. Fred has a full-time job and files a Schedule E (active participation) for the rental house. Which of the expenditures must be depreciated rather than deducted as an expense on his Schedule E? I. Replaced a screen in a window II. Replaced the air conditioning system III. Built a swimming pool IV. Installed a new water heater V. Hired a lawn service to cut the grass

II, III, IV Items I and V are expenses. The others are improvements even though II says replace and IV says install. There is no list of what are expenses and what are improvements. The dollar amount determines the difference. A screen may be a $100. The air conditioning system could be $3,000-$10,000. These must be capitalized, added to the basis of the property and depreciated.

Is alimony earned income?

If from before 2019, yes If not before 2019, no

Tax-Sheltered Annuity

Retirement savings plan that allows employees of tax-exempt organizations and self-employed people to invest pretax dollars to build retirement income. Designed to provide consistent payouts over time and act as a reliable source of income in retirement.

Partnership keys

Risk-free business Keogh 100% of medical, dental, and LTC insurance is deductible for owner Lack of continuity Losses up to basis Deduct half of SE tax

Risk Transfer

Shifting of a pure risk from one party to another. Ex: purchase of an insurance policy

Option Time Value

TV = Premium - IV Ex: $8 premium, $3 IV --> $5 TV

SEP IRA (contribution, vest, SS, eligibility)

Up to 25% contribution for owner, up to 18.59% for self-employed Account immediately vested Can be integrated with Social Security 21+, paid at least $650, worked 3 of last 5 years to be eligible

Stock Bonus Plan (deduction, contributions, SS)

Up to 25% employer deduction Flexible contributions 100% of contribution can be invested in company stock ESOP cannot be integrated with Social Security or cross tested

Risk Control or Risk Financing 1 Risk Avoidance 2 Risk Retention 3 Risk Reduction 4 Risk Diversification 5 Risk Transfer

1 Control 2 Financing 3 Control 4 Control 5 Financing

CASE T.J., upon separation from service, elects a lump sum, in-kind distribution of all employer stock from the qualified plan. The basis of the stock is $200,000, and the FMV of the stock is $500,000. The stock is placed in his personal brokerage account. Question 1 What is the tax ramification of the NUA distribution? A. No tax until he sells the stock; B. Ordinary income tax on $200,000 plus a 10% penalty; C. Ordinary income tax on $300,000; D. Ordinary income tax on $500,000 plus a 10% penalty; E .Ordinary income tax on $200,000 Question 2 What happens if T.J. sells the stock for $700,000 6 years in the future at age 60? HINT: Does the information in Question 2 affect Question 1? Yes, it is possible. But here it does affect Question 1. This would only occur in a mini case or full case. Very low probability, but possible. A. The $200,000 is LTCGs.; B. The $500,000 is LTCGs.; C. The $700,000 is LTCGs.; D. The $200,000 is ordinary income. Question 3 If T.J. dies before separating from service, and his qualified plan is passed to his son, can the son take the NUA? A. Yes; B. No, NUA ceases at death since the assets will be stepped up.; C. Yes, but the son may have a 10% penalty.

1. B. Ordinary income tax on $200,000 plus a 10% penalty Unless the plan specifies separation from service after attaining age 55 or he is age 59½, the 10% penalty applies to basis only. He has to be age 54 because of Question 2. NOTE: He takes a lump sum distribution. He does not sell the stock so only the basis is affected. 2. B. The $500,000 is LTCGs. 3. A A retirement plan cannot receive a step up. NUA would still be available to his son without a 10% penalty (beneficiary).

Anything above estate exemption taxed at

40%

GSTT tax rate is

40% after the 40% tax on estate

A $1,000 bond originally issued at par matures in exactly 10 years bears a coupon rate of 8%(payable semiannually), and a market price of $1,150. The indenture agreement provides that the bond may be called after five years for $1,050. What is the yield to call?

5.42 CLR->2 P/Y->-1,150 PV->1,050 FV->80/2 PMT->5xP/Y N->CPT I/Y END MODE

A $1,000 bond originally issued at par matures in exactly 10 years bears a coupon rate of 8%(payable semiannually), and a market price of $1,150. The indenture agreement provides that the bond may be called after five years for $1,050. What is the yield to maturity?

5.98% CLR->2 P/Y->-1,150 PV->1,000 FV->80/2 PMT->10xP/Y N->CPT I/Y END MODE

What % of agi can you deduct for charitable contribution if gift is cash and stock? If cash? If stock?

50% of agi Up to 60% of agi Up to 30% of agi

William Smith, a very conservative investor, is seeking investment advice. He wants to squeeze out as much current yield as possible from his bond portfolio with a barbell approach. He believes by the end of next year the Federal Reserve will begin to raise interest rates. How should a CFP® professional respond? A. It is a good idea if you anticipate higher interest rates. B. It is a good idea if interest rates stay at the current level. C. It is not a good idea if interest rates stay at the current level. D. It is not a good idea because short-term bonds are practically paying nothing.

A *Barbells are implemented to take advantage of a rising rate environment. Be careful sometimes barbell is tested. You may not agree with the answer.* Answer C, although true, is not the best answer. Mr. Smith believes interest rates are going higher. Answer B is false. In this case the yield curve will steepen and the longer-term bonds would suffer. Answer D is true currently, but again, it does not answer the question. The basic premise of a barbell strategy involves overweighting longer-term bonds, which are more sensitive to interest rate hikes, as well as overweighting shorter-term bonds on the other end of the curve. The strategy also underweights exposure to intermediate-term, 5- to 7- year bonds. Thus, a hypothetical barbell is created.

A client calls their CFP® professional. He heard about a mutual fund. He asks you if the mutual fund is a wise investment and if he should buy it. The planner researches the mutual fund and find out the following: Alpha = -7 Beta = .4 R2 = 80% SD = 10% T-bills = 4.5% What would a CFP® professional recommend? A. A CFP® professional tells the client not to buy the mutual fund because the alpha is negative. B. A CFP® professional tells the client to buy the mutual fund because the beta is low. C. A CFP® professional tells the client to buy the mutual fund because the Sharpe index will be high. D. He should buy the fund because the standard deviation is low.

A *When the R2 is high, look for the alpha answer.* Alpha is negative, your suggestion would be to not buy the fund.

Mr. and Mrs. Dodd are in a 12% marginal income tax bracket. Mr. Dodd wants to sell $15,000 of stock with a basis of $10,000 held for 13 months. He feels that the sale will not change his 12% marginal income tax bracket. How much additional tax will he have to pay? A. $0 B. $250 C. $750 D. $1,500

A 0% of $5,000 (long term capital gain when the client is in a 12% marginal income tax bracket) The question clearly states the sale will not change his 12% marginal income tax bracket.

Which of the following plans cannot effectively be integrated with Social Security? A. 401(k) plan (no match or company contribution) B. Money purchase plan C. Stock bonus plan D. Defined benefit plan

A A 401(k) with no match or profit sharing cannot be integrated with Social Security. Profit sharing plans can be integrated. It does not ask whether or not a profit sharing 401(k) can be integrated. But, Answer A does indicate it is a pure 401(k) with no match or profit sharing contribution. There is nothing to integrate in the 401(k) plan. So Answer A is out, but Answers B and D can be integrated. Stock bonus plans can be integrated.

Frank, currently age 44, believes that the business cycle is about to turn sharply and that an 8% inflation rate is a necessary assumption in the construction of his retirement plan. His CFP® certificant, strongly believes that inflation is and will continue to be substantially lower averaging between 3 and 4 percent over the long run. Which inflation rate from the choices below should the CFP® professional reject first? A.The current year's inflation rate B.Frank's inflation rate assumption C.The 10-year average inflation rate per the CPI D.The 100-year average inflation rate

A A one-year inflation rate does not constitute an inflation rate for a long-term goal achievement. Remember that the client and planner must mutually agree on plan assumptions. While the client's 8% assumption may not end up as the one used in the plan, it should be considered and discussed based on his concerns. It might make sense for the planner to run the numbers using both his personal inflation assumption and run them again using the client's so the client can see that his assumption would probably be less workable.

Ted has to file for Chapter 7 bankruptcy due to extreme credit card debt balances. Which of the following is an exemption or exception? A. Cash value of his life insurance policies B. A new Lexus C. Very expensive home D. Gold coin collection

A Also expensive car ownership is a debatable asset. A very expensive home is also debatable because most states have dollar limits. Only some limited amount of personal property applies. No dollar amount is indicated for the gold coins. But Answer A definitely applies

Jake and Louise Wallen have been margining their portfolio. They have active income of $100,000. Their portfolio has $25,000 of investment interest. They have portfolio income of $40,000. However $30,000 is from qualified dividends. How should they treat the qualified dividends? A. Leave them alone as qualified dividends B. Elect out of the qualified reduced rates to get an investment income deduction C. Deduct the other $10,000 of portfolio income since it is not qualified and just use investment interest deduction D. Just take the standard deduction

A Although Answer C could be true, there is actually no mention of what the other portfolio income is. It could be long-term gains. Remember the question is only asking about the qualified dividends. There is no mention of any other itemized deductions. Therefore the investment income deduction is less than the standard deduction and they have to pay ordinary rates on the dividends. Although Answer D is true, it does not answer the question (qualified dividends).

Terrie Cross and Brenda Davis have decided to close their business. They have cross-purchase life insurance policies in force. Both Terrie and Brenda are married. How should they handle the split-up of their policies? A. Each should purchase her own policy from the other person. B. No change. C. Terrie should transfer Brenda's policy to Brenda's husband, and Brenda should transfer Terrie's policy to Terrie's husband. D. Same as Answer C, but to opposite husbands.

A Answer B makes no sense regarding the situation. Answers C and D trigger transfer for value. The insured can always buy their own policy and not trigger transfer for value.

Mr. Treble is interested in a particular stock. In reviewing the stock he found a report indicating that the correlation of the stock to the S&P 500 Index was only .5. What does that reveal about the stock? A. It will have a lower beta than the S&P 500 Index. B. It will have a higher volatility than the S&P 500 Index. C. Nothing, it is not material in that an individual stock cannot be explained by the S&P 500 Index D. It is 50% less volatile than the S&P 500 Index

A Beta is directly related to correlation. A correlation of less than 1 would mean a lower beta. Correlation is not exactly the same as the volatility percentage. Beta is the volatility percentage. Correlation is used in the beta formula. The question really relates to understanding the formula. Can a single stock have a meaningful beta? Yes, GE and the S&P 500 tracked almost perfectly together for many years. Look at the formula sheet and you will see the connection.

Which of the following statements are true about Coverdell ESA accounts? I. The beneficiary must be under 18 in years when contributions are added to the account. II. Contributions are treated as a gift of a present interest from the contributor to the beneficiary. III. Qualified expenses include religious elementary school expenses. IV. Permissible investments include individual stocks and bonds. A. All of the above B. I, II C. I, II, IV D. II, III E. III, IV

A Coverdell can cover various elementary school expenses.

Which of the following statements about disability payments and benefits are true? I. Employer contributions to employee group disability insurance plans are deductible. Benefits will be taxable income to the employee. II. When an employee pays for an individual disability policy, or the plan is contributory, the employee pays for the plan with after-tax dollars. Then the benefits will be tax-free to the employee. III. The employer uses an executive bonus (Section 162) to purchase an individual disability policy for an employee. The premium is deductible by the employer as a bonus, and the benefits are tax-free to the employee. IV. 2% or greater owners of S corporations always get tax-free disability benefits. A. All of the above B. I, II, III C. I, II D. II, III E. III, IV

A Employer contributions for group disability insurance will result in no taxable income to the employee. However, the payment of benefits will result in taxable income (Answer I). If the plan is contributory (executive bonus - Section 162) and the employee is charged with the insurance premium (bonus), then the benefits are tax-free. Please refer to the Live Review Insurance session. Yes, Answer IV is true when it says more-than-2%-owners.

Can 529 plans pay for K-12 education expenses beyond tuition? A. No, it can only be for tuition. B. It depends on whether it is a private or public school. C. Yes, if the child has a physical disability. D. No, 529 plans are only for college.

A Even a 529 ABLE is limited to tuition only for grades K-12.

Howard Hughes, age 57, has decided to quit work and travel extensively while his health remains good. Howard's family history is not good with parents, brothers, and sisters all dying early due to medical problems. He feels that since he has no family responsibility he will use his $400,000 in his 401(k) as follows: - Roll $100,000 into an IRA. - Leave $100,000 in the existing 401(k) because the company he is working for is willing to give him a leave of absence. - Take a $200,000 distribution with 20% paid to IRS and $160,000 paid to him. He expects to pay taxes on the $200,000 but no 10% penalty. Can he do this? A. Yes B. No C. Only leave $400,000 in the 401(k) D. Only take the $200,000 distribution but pay a 10% penalty E. Only a distribution of the whole $400,000 from the 401(k) to avoid the 10% penalty

A He is doing a separation of service after age 55. He can do all three. You might argue that choice 2 says the company is willing to give him a leave of absence. However, the question says he is going to quit. Even if he quits, he can leave money in the 401(k). The question is asking what he can do, not what he should do.

Rod Hotshot bought a stock on a tip. His basis was $3,200. (His basis included $200 in commissions.) He sold it for $10,000 after commissions were paid. Rod is retired and is in a 12% income tax bracket. How much tax would Rod owe if he held the stock for more than one year based on a 12% income tax bracket? A. $0 B. $350 C. $1,020 D. $1,050 E. $1,500

A He is in a 12% tax bracket; the capital gains rate is zero.

Larry is disabled due to a job-related injury. He is being paid benefits under workers compensation, Social Security, and his private disability plan (employer paid). Which of the disability benefits could be taxable? I. Workers compensation disability benefits II. Social Security disability benefits III. Private disability plan benefits paid by his employer (salary continuation) A. All of the above B. I, II C. I, III D. III

A If his MAGI is above certain levels, the Social Security benefits could be taxable. The private disability plan benefits are taxable because the employer paid the premiums. It does not say it was a taxable bonus. Workers compensation may be taxable when that same person gets Social Security disability. If you know II and III are true, then Answer A is the only possibility.

What are the purposes of immunizing a bond portfolio? I. To reduce the bond portfolio's market losses II. To earn a specific rate of return from the bond portfolio over a given period of time, regardless of what happens to market interest rates III. To offset interest rate risk with reinvestment rate risk IV. To avoid buying zero coupon bonds A. I, II, III B. I, II, IV C. II, III, IV D. II, III E. III, IV

A If interest rates rise, interest rate risk causes the value of the bonds to drop, but the client can earn more on coupon payments that are reinvested. If interest rates decline, interest rate risk causes the value of the bonds to rise, but the client will earn less on coupon payments that are reinvested. The typical method of immunizing involves assembling and appropriately managing a diversified portfolio of bonds. Choice II works because you are buying bonds to hold until maturity. You know the YTM of your portfolio when you make the purchase.

Mr. and Mrs. Tate have two children, ages 2 and 4. Their day-care expenses are $10,000. Both Mr. and Mrs. Tate work. What amount of child-care credit will they receive? A. $1,200 B. $2,000 C. $3,000 D. $6,000

A It asked for child-care credit, not the child tax credit. ($6,000 x 20% = $1,200) The maximum expenses that can be used in figuring the credit is $6,000. They only get 20% of the expenses as an actual credit.

Tom Jennings has asked about 1040 income. Which of the following is considered 1040 income? A. Long-term capital gains B. Private placement municipal bonds interest C. IRA rollover into another IRA D. Deductible IRA contribution

A Long-term capital gains are taxed differently but are 1040 income. Private placement income affects AMT but not 1040 income. The IRA is a direct rollover. The IRA contribution is an adjustment to income. It is not income.

Into which of the following plans could a sole proprietorship with no full-time employees put the maximum retirement money away for the owner? A. Keogh defined benefit B. SIMPLE C. SIMPLE 401(k) D. SEP E. Solo 401(k)

A Self-employed persons can have a Keogh defined benefit plan. It would allow up to the maximum defined benefit to be deposited each year. There is no contribution limit, only a benefit limit. Under some circumstances, the contribution could be $400,000 or more. There are no 12.12% or 18.59% limits on DB plans for self-employed owners under DB type plans. They only apply to DC and SEP plans.

A premature distribution penalty tax applies to which one of the following IRA distributions? A. A distribution made to the owner ($10,000 lifetime limit) for the purchase of a primary residence B. A distribution made to the owner for qualified higher education expenses furnished to the owner C. A distribution made after the death of the owner D. Medical expenses in excess of 7.5% of AGI

A The distribution must be for the purchase of a first home, not necessarily a primary residence.

Which of the following is (are) subject to self-employment tax? I. Distributive share of limited partnership operating income II. Wages from an S corporation III. Distributive share of general partnership operating income IV. Interest and dividends from investments A. III B. I, III C. II, III D. I, IV

A The general partnership operating income is self-employment income. By definition, the other items of income are not subject to the self-employment tax. They are forms of unearned income. S corporation wages are subject to FICA, not self-employment taxes.

Which investment provides the maximum leverage and the maximum hedge against inflation? A. Improved land B. Common stock C. A mortgage REIT D. An LP

A The improved property can be purchased using a bank loan. Improved property is a hedge against inflation. Common stock generally loses value during inflationary times. Mortgage REITs are highly leveraged but do poorly during times of high inflation since underlying can only be raised as leases renew. LP? What kind of LP?

Steel Building Partners (a general contractor) erects multi-story buildings. All construction employees are insured by workers' compensation and group disability insurance. Construction sites have elaborate safety nets. Which answer includes two methods of handling risk being used by Steel Building Partners? A. Risk transfer and risk reduction B. Risk retention and risk avoidance C. Risk avoidance and risk diversification D. Risk sharing and risk transfer

A The insurance is a risk transfer, and the use of nets is a risk reduction technique.

Bob has retired. His company had an ESOP. Stock with a basis of $50,000 was contributed to it. Stock with a market value of $125,000 was distributed at Bob's retirement. Six months after retirement, Bob is selling all the shares for $150,000. What is Bob's tax situation? A. $50,000 was taxed as ordinary income at the time of retirement; $75,000 will be taxed at LTCG rates at the time of sale, and $25,000 will be taxed at STCG rates at the time of sale. B. $150,000 will be taxed at LTCG rates at the time of sale. C. $100,000 will be taxed at LTCG rates at the time of sale. D. $50,000 was taxed as ordinary income at the time of retirement; $100,000 will be taxed at LTCG rates at the time of sale.

A The net unrealized appreciation is taxable as long-term capital gain to the recipient when the shares are sold, even if sold immediately. If the recipient holds the shares for a period of time after distribution, any additional gain (above the net unrealized appreciation) is taxed as long or short-term capital gain, depending on the holding period. The $25,000 gain between distribution ($125,000) and the sale ($150,000) is STCG because only 6 months went by.

What is not an attribute of a Coverdell ESA? A. Deductibility of contribution B. Contributions are only $2,000 per year per student C. Expenses are not limited to qualified higher education expenses D. Qualified elementary education expenses can be paid tax-free

A The others are attributes because the contribution is not deductible. Yes, Answer B is a weak attribute.

Which type of entities would buy preferred stock paying a high dividend? I. Pension plan II. Individual in a 12% bracket III. Regular C corporation with excess funds to invest IV. Spouses in a high 37% tax bracket A. I, II, III B. I, II, IV C. I, II D. II E. II, IV

A The pension plan would buy for income. The individual would buy because they would pay no tax (dividends in a 12% bracket are subject to 0% tax). Regular C corporation was an answer to a prior question. Test taking: If II and III are correct, the answer has to be A. Answer IV will result in the dividend being taxed at 20% (37% bracket). Answer IV is not a bad answer, but Answers I, II, and III are better because of the tax results.

If all the employees get $150,000 of group term coverage, then which of the following is true? A. Only the cost of up to $50,000 is tax-free. B. The cost of the $150,000 is tax-free. C. The plan is discriminatory in favor of the lower paid employees. D. The cost will be fully taxable.

A The plan is not discriminatory. Everyone gets the same amount of coverage. The first $50,000 is tax-free.

Mr. Sims purchased a $500,000 life policy 35 years ago (at age 27) with a single premium of $50,000. The contract cash value has grown to $110,000. He has decided to surrender the contract this year. Which of the following is true? A.$50,000 of the $110,000 will be income tax free; the remaining $60,000 will be subject to tax at ordinary income tax rates. B.$50,000 of the $110,000 will be income tax free; the remaining $60,000 will be subject to tax at capital gains rates. C.$60,000 will be subject to tax at ordinary income tax rates plus a 10% penalty. D.$110,000 will be subject to tax at ordinary income tax rates.

A The policy is not a MEC; therefore, the cash value in excess of basis ($50,000) will be subject to tax at ordinary income tax rates, but not the 10% penalty. 2022 - 35 years ago is 1987. It was purchased before 1988. Single premium policies before 1988 are not considered MEC policies. They were grandfathered. In addition, Mr. Sims is over 59½, so there would be no 10% penalty if the policy was a MEC.

Charlie was granted an incentive stock option (ISO) four years ago when the FMV and option price was $20 per share. Charlie exercised the option two years ago when the stock was trading at $30 per share. If he sold the stock today for $35 per share, which statement(s) is/are not true? A. Charlie was taxed for the $10 per share gain at capital gains rates when he exercised the option (no substantial risk of forfeiture). B. There are no regular income tax consequences when the ISO is granted or exercised; Charlie will pay capital gains when the stock is sold at a gain. C. If Charlie sells the stock for $35 per share, he will be taxed at capital gains rates on the $15 gain per share. D. At the time of exercise, $10 per share was an add-back item.

A The stock was held for the two-year/one-year holding period. It qualifies for LTCG and was an AMT add-back. The question does not indicate any AMT was paid, therefore the basis is the cost of the shares.

Real Rate of Return (Inflation adjusted interest rate)

Nominal rate of return adjusted for inflation = [(1+After-tax Return / 1+Inflation Rate)-1]X100

What are two considerations that a financially well-off individual should consider when buying a home? A. Home cost and maintenance costs B. Individual marginal tax bracket and time period of potential ownership C. Large capital gain exclusion on sale and deduction for property taxes D. Mortgage rates and home cost

B Remember it is asking about two considerations. The key in Answer B is the time period of potential ownership. All the other answers are gain or cost related, not time related. For someone well-off, cost may be less of a major concern.

Terry is a former teacher who now works for Software, Inc. She has a personal IRA, a 403(b) from her teaching career, and now a 401(k) at Software, Inc. Terry will turn 72 in April of this year. She plans to continue working for Software, Inc. until she turns age 75. From which plans must she take RMDs by April 1st of next year to avoid the 50% penalty? A. Only the IRA B. The IRA and 403(b) C. All three plans D. None of the plans

B She does not need to take an RMD from the current 401k.

How to calculate RMD

Account balance / Life expectancy factor

Why use GRIT, GRAT, or GRUTs?

Amount paid to grantor over life of trust is subtracted from value of initial transfer to reduce amount subject to gift tax If grantor outlives the predetermined number of years of scheduled payments, assets in trust can be accumulated without the accumulated value becoming part of the grantor's taxable estate If grantor dies prior to the expiration of the predetermined number of years for which payments are scheduled, the assets owned by the trust will all be considered part of the grantor's taxable estate

John, age 69, and Mary, age 66 (married) have been collecting Social Security benefits of $1,500 for him and $750 for her per month. If he dies, what will happen to her benefits? A. She will get nothing. She gets ½ of his benefits. B. She will get his current benefit. C. She will get $1,250 per month. D. She will get $2,250 per month.

B She will get the greater of her benefits or 100% of his benefits.

Cindy purchased a whole life policy 15 years ago. The annual premium is $3,000 a year. She used the dividends to reduce the premium each year. (Dividends paid were $15,000 over 15 years.) If she surrenders the contract for $40,000, what amount of the proceeds will be taxable? A. $0 B. $10,000 C. $20,000 D. $30,000

B Surrender value - Premiums billed = 40k - 30k (30k= 45k - 15k dividends)

Mrs. Tuttle spent 110 days in a skilled care facility. She qualified for her stay under Medicare. The facility cost was $394.50 per day and the Medicare specified amount is $194.50 per day. How much did Medicare pay? NOTE: The Medicare specified amount will be given on the exam if they ask this type of calculation. A. $16,000 B. $23,890 C. $31,560 D. $39,450

B Medicare will pay the first 20 days in full @ $394.50 plus $200 (everything above $194.50) for the next 80 days. There is no coverage after 100 days. 20 days @ $394.50 = $7,890 80 more days @ $200 = $16,000 TOTAL $23,890

Barry retired a few years ago from Belmont, Inc. at age 65, leaving the balance in his 401k plan. He is turning age 72 in March of next year (2023). He is planning to return to work at Belmont toward the end of next year on a full-time basis. Out of which of the following accounts does Barry have to take a mandatory distribution by April 1st of 2024? I. SIMPLE IRA II. SEP IRA III. Roth IRA IV. Traditional IRA V. Belmont 401(k) plan (account from retirement at age 65) A. All of the above B. I, II, IV, V C. I, II, III D. I, II, IV

B A minimum distribution is required for the year in which the participant attains age 72. The way the question is phrased, Barry will still be retired when he reaches age 72 in March of 2023 He will be back to work at the end of the year and may not be eligible to participate in the 401(k) until next year. He will not be working for Belmont when he turns 72. Tough. He will be marked by the plan as terminated when his 1st RMD is calculated, thus requiring an RMD for the 1st year. He may not need to take subsequent RMDs as he remains employed.

Mrs. Jackson died and left $150,000 to her grandson (a skip person). She had already used $12,060,000 of her GST tax exemption. How much GST tax will be due at her death in 2022 if her maximum estate tax rate is 40%? HINT: The estate tax offsets the GST tax like in the practice questions. A. $16,000 B. $36,000 C. $90,000 D. $120,000

B No annual exclusion is available at death. $150,000 x 40% = $60,000 gift tax due, then $150,000 - 60,000 = $90,000 $90,000 x 40% = $36,000 GSTT

Ken, age 65, is eligible for Medicare. He would like to delay enrolling in Medicare until he retires. He asked you if he can continue to be a participant in the company's HSA (over 20 employee plan). How should a CFP® professional respond? A. Because he is eligible for Medicare, he can no longer be eligible for the company's HSA. B. Yes, he can continue to participate as long as he does not enroll in Medicare A and B. C. Yes, but only if he does not enroll in Medicare Part B. D. Medicare will be automatic when he turns age 65.

B An individual is "entitled to benefits under Medicare" only when the individual is both eligible for and enrolled in Medicare. Enrollment in either Medicare Part A or Part B will disqualify an individual from contributing to an HSA, but mere eligibility does not preclude HSA contributions. NOTE: You are not automatically covered for Medicare at age 65. You must enroll by contacting Medicare. The actual ruling is "if you are not collecting Social Security when you become eligible for Medicare you must enroll through Social Security."

Hank Wellington, age 60, has been totally disabled for the past year. He has recovered, but his doctor only wants him to work part-time for 6 months. The doctor indicated 20 hours per week or about 4 hours per day. Based on the information, can Hank get disability benefits? A. No, he will no longer be totally disabled B. No, unless his policy has a partial or residual rider C. With a partial benefit rider he will get a proportional amount of income lost and payable for the same duration as total disability D. With a residual benefits rider he will get 50% of the total disability benefits for a period of time

B Answer B is a more complete answer than Answer A. There is no indication whether there is a rider in the policy or not. Answer C is the residual rider Answer D is the partial rider. Answer B is like picking both Answers C and D.

Which statement is not true about an existing Health Savings Account (HSA)? A. An HSA may receive contributions from any person, including an employer or family member, on behalf of an eligible person. B. Contributions, other than employer contributions, are deductible on the eligible person's individual tax return only if that person itemizes. C. Employer contributions are not included in the eligible person's income. D. Both the employer and eligible person may contribute to the HSA in the same tax year.

B Answer B is wrong because the contributions are deductible on the eligible person's tax return even if that person does not itemize. They go on the front of the 1040 (adjustments to income). Deductions are for AGI not from AGI (itemized and standard deduction).

What is a similarity between financial advice and financial planning? A. The integration of various financial subject areas B. The fiduciary standard C. The length of time of the recommendation D. The impact on the clients' overall financial health

B Answers A, C, D are all factors that separate planning from advice. The fiduciary standard applies when giving either advice or planning, but it is still important to know the difference. A CFP® professional must act as a fiduciary while giving financial advice or financial planning.

Which of the following assets get a step-up in basis when the client dies? A. A $100,000 CD now worth $105,000 (bought 5 years ago) B. A municipal bond purchased at a discount ($95,000) two years ago now worth $100,000 C. Stock purchased 6 months ago for $50,000 that has a $10,000 STCL D. An annuity purchased as an investment 10 years ago for $100,000 now worth $115,000

B Answers A, C, and D are ordinary income property. Bonds are investments, like stocks. They get a step-up in basis at death. Yes, income from a bond is ordinary income. The stock has a capital loss, not a gain.

Randy Watkins has been contributing $200 per month to his annuity for 20 years. The annuity is part of his retirement planning. He is single and is planning to take a 20 year pure life annuity payout of $660 per month when he retires. How much of the annuity payout will be excluded? He believes his tax bracket in retirement will be 24%. A. 28% B. 30.3% C. 9.7% D. 72%

B Basis/Payout = 200x12x20/660x12x20 = 48,000/158,400 = 30.3% You could have done it simply by just dividing $200 by $660

Parents ($120,000 MAGI filing jointly) with one child in the first year of college should choose which of the following credits if they pay tuition of $10,000 in the current year? A. A Coverdell ESA of $2,000 B. An American Opportunity Credit of $2,500 C. A Lifetime Learning Credit of $2,000 D. An UTMA gift of $10,000

B Both Answers B and C are correct, but Answer B is greater and should be the parent's first choice. The question is asking about credits; Answers A and D are not credits.

Which of the following is/are considered conduit income? I. K-1 income from an S corporation II. K-1 income from a partnership III. Dividend income from a corporation IV. DNI from a trust V. K-1 loss from active participation in real estate A. I, II, III B. I, II, IV C. II, III D. II, V E. III

B DNI is distributed net income. A simple trust uses the conduit principal (DNI). Answer V would have been correct if it said K-1 income, but it says loss. Dividends from a corporation are after-tax distributions.

Which of the following is true about Coverdell ESAs? A. Multiple $2,000 contributions can be made on behalf of one beneficiary per year by multiple individuals (not both parents). B. Expenses such as tuition can qualify even when they are for graduate school (to age 30). C. Contributions to the ESA are tax-deductible. D. All earnings must be paid out at age 18. E. Contributions may only be made by a parent.

B Earnings must be distributed when the beneficiary reaches age 30. The aggregate contribution on behalf of a beneficiary cannot exceed $2,000.

A client has a $1,000,000 portfolio consisting of the four stocks listed below: 1. $200,000 ABC @ 1.5 beta 2. $250,000 DEF @ 2.0 beta 3. $150,000 GHF @.5 beta 4. $400,000 JKL @ 1.2 beta What is the weighted beta of the portfolio? A.1.3 B.1.355 C.1.42 D.1.46

B Everything is a percentage of $1,000,000. 1. 20% x 1.5 = .30 2. 25% x 2.0 = .50 3. 15% of .5 = .075 4. 40% of 1.2= .48 TOTAL 1.355

Mr. Simms wants to buy a stock on the Japanese market. He wires $100,000 to Japan and buys the stock when the currency rate is 125 yen to a dollar. In the next year, the stock rises by 50%, and he sells it. The proceeds are then converted to dollars when the currency rate is 150 yen to a dollar. What is his return in U.S. dollars? A. 12.5% B. 25% C. 37.5% D. 50% E. 80%

B First, convert dollars to yen at 125 yen to the dollar 100,000x125 = 12,500,000x1.5 = 18,750,000 Now convert back to dollars 18,750,000/150 = 125,000 -> 25% return

A worker will be entitled to Social Security disability benefits if which of the following is true? I.has been disabled for 12 months or is expected to be disabled for at least 12 months or has a disability that is expected to result in death II.is NRA (normal retirement age) or over III.is insured for disability benefits IV.has filed an application for disability benefits V.has completed a 5-month waiting period or is exempted from this requirement A. All of the above B. I, III, IV, V C. II, III, IV D. III, V E. IV, V

B For disability, he/she must be under NRA (normal retirement age). Answer V is correct because of the word yes, it starts in 5 months if it is known that death will result. If not, then there is a year waiting period. At that time the 7 months in arrears is paid

Harry Long established a business many years ago which he sold for $15 million net of taxes. That, plus various investments, pension plans, and Roth IRAs have built his net worth with his wife's assets to $32 million. The first thing he and his attorney did was equalize the assets between Harry and his wife Martha. Martha now has $16 million in her name. Now Harry is working with his attorney to set up a trust in which Martha could only invade the corpus if she had dire circumstances. Which type of trust should a CFP® professional recommend? A. A dynasty trust for Martha B. An accumulating B trust with HEMS provisions for Martha C. An irrevocable trust with income provisions for Martha D. A QTIP trust with HEMS provisions for Martha

B Harry will want to take advantage of the $12.06 million estate exemption. By restricting the "B" trust to accumulating and giving the trustee discretion over any distributions (HEMS), this trust meets his objectives. In both Answers C and D, she will get income distributions. This will add to her estate and potential tax. The "B" trust, no matter what it grows to, will pass estate tax free. Yes, the irrevocable trust will pass estate tax free, but it does not have any HEMS provisions. The QTIP passes estate tax free at his death but then is subject to estate tax at her death. The dynasty trust is for his children and grandchildren not his wife Martha.

Richard Daley, age 35, is employed by ABC, Inc. He makes $130,000 per year. What type(s) of retirement plan should the CFP® professional recommend to him? A. SEP at 18.59% B. A Roth IRA and/or a deductible IRA C. A Roth IRA and/or a nondeductible IRA D. A Roth IRA

B He is an employee. He is not self-employed so SEP is not an answer. Nothing indicates ABC, Inc. has a retirement plan. Unless the question indicated the employee is active, then the employee is not active. He is not subject to the IRA phaseout rules. His income is under Roth phaseout. Yes, Answer C is also an answer, but why not choose a deductible IRA?

Arthur is in a 35% income tax bracket. He has decided to buy a $40,000 car with cash. He needs to sell an investment to raise cash. Which one of the following assets would generate the least amount of tax liability if sold? NOTE: LTCGs rate is 15% for this question A. An annuity worth $30,000 with a basis of $28,000 B. A stock bought 6 months ago for $27,500, now worth $30,000 C. A stock bought 13 months ago for $24,000, now worth $30,000 D. A baseball card bought 2 years ago for $26,000, now worth $30,000

B In regards to the annuity, you cannot assume he is over age 59½. The question must tell you the person is over 59½. You must assume it is a deferred annuity because it is worth more than he bought it for originally. It cannot be an immediate annuity. The tax is calculated as follows for each answer: A. $2,000 ordinary income at 35%, plus 10% penalty on $2,000 = $900 (under 59½) B. $2,500 STCG at 35% = $875 C. $6,000 LTCG at 15% = $900 D. $4,000 at 28% = $1,120

For the current tax year, Bob Pearson, an individual taxpayer with a $100,000 AGI, has $12,000 of investment interest expense and $8,500 of investment income. He also has financial advisor fees of $1,500. How much investment interest expense, if any, may Bob deduct in the current tax year? A. $7,000 B. $8,500 C. $10,000 D. $11,300 E. $12,000

B Investment interest expense is deductible up to the amount of net investment income. Net investment income is $8,500. The financial adviser fees are no longer deductible.

Barbara and Kathy are 50/50 owners of BK, Inc. When they started BK, Barbara and Kathy had the company take out two $1,000,000 key-person term life insurance policies. Over the years, the company has paid $5,000 in premium on Barbara's policy and $4,000 in premium on Kathy's policy. Barbara and Kathy have decided to use the policies to do a cross purchase buy-sell. If Barbara buys Kathy's policy for the premium paid by BK, what will be the result if Kathy dies within one year? A. Barbara will get $1,000,000 income tax free (term insurance exclusion). B. The policy will be subject to transfer for value. C. The policy will be included in Kathy's estate (3-year rule). D. The policy will increase the value of BK, Inc. by $1,000,000 (3-year rule).

B It does not matter whether the policy is term or permanent insurance. Transfer for value makes the policy income taxable above basis. The 3-year rule doesn't apply when transfer for value occurs. Only Barbara can buy Barbara's policy, and only Kathy can buy Kathy's policy. This is a corporation entity, not a partnership entity. A partnership is an exception to the transfer for value rules.

Mrs. Barnes, age 75, has no family members that she wants to inherit her money. Her doctors have given her a 3-year life expectancy. Her assisted living facility gave her a one-time payment offer based on her life expectancy. So she currently has no monthly expense. She is getting the maximum Social Security benefit (husband's benefit) and income from her investments. She wants to give to the Humane Society. If she gives $25,000 in cash and $50,000 in stock (basis $15,000), how much can she deduct this year if her AGI is $80,000? A.$35,000 B.$40,000 C.$49,000 D.$75,000

B It does not matter whether you use the cash first ($25,000) and then the stock (30% AGI) to calculate or not. The cash is eligible up to 60% of AGI. But because it is a combination of cash and stock the 50% of AGI ($80,000) rule is the maximum. You may have selected Answer C, but $49,000 exceeds 50% of AGI.

Mrs. Todd, age 70, has been working since her husband died 20 years ago. The jobs have always been low paying but at least the last one provided a pension of $1,000 a month since she waited until age 70 to retire. Social Security at 70 provides her with another $1,500 per month. She feels she needs another $4,000 per month of income. She is debt free with a nice basic house near her daughter. If inflation is averaging 2.5%, what kind of yield would she need to attain based on $700,000 of investable assets without using up principal? A. 2.57% B. 6.86% C. 7.12% D. Without knowing her life expectancy the question is not solvable

B It really is just basic math since the principal and income remain constant; this is not a time value of money question. The $4,000 is the needed income. There is no mention of solving for todays dollars so inflation does not matter. This is not TVM, only 2 variables are here. 12 x 4,000 / 700,000 = 48k/700k = 6.86%

Tom, age 51, works for a not-for-profit organization. He contributes $20,500 annually to a 403(b). Tom's wife, Melinda (age 48), works for another not-for-profit organization. She contributes $5,000 annually to a 457. If he earns $110,000 a year and Melinda earns $50,000 a year, how much can he and she contribute and deduct to a regular IRA in 2022? A. $500 B. $6,000 C. $7,000 D. $12,000 E. $14,000

B The 403(b) retirement plan makes him an active participant. The nongovernmental 457 does not trigger active participation for her. She is not active. Their combined AGI is $134,500 after their deferrals. Her income counts. She can contribute to a deductible IRA under the spousal rules (less than $204,000). He is subject to phaseout ($109,000 - $129,000). He is active.

Mr. Phillips is a widower. He inherited a substantial amount of money from his deceased wife (second marriage for both). Now he is faced with $5 million in estate taxes should he die. He would like to give the money to charity but receive income for the remainder of his life. He is concerned because his wife wanted all her money to eventually pass to her children. What should a CFP® professional suggest? A. Do a charitable lead trust with the children as the ultimate beneficiary B. Do a CRAT with fixed income to him for life and a wealth replacement trust C. Do a CRUT with variable income for him for 20 years and the remainder paid out over the lives of her children D. Give the money to her children

B The CRAT accomplishes his first purpose: income to him for life. The wealth replacement trust is a life insurance policy on Mr. Phillips, normally equal to the amount that was supposed to be paid to the children and paid for with the charitable tax deductions and some of the income from the CRAT. In Answer B, the children get the life insurance proceeds.

What is the biggest risk of establishing an ILIT with an existing insurance policy? A. The trust purchases a life insurance policy. B. The existing policy will be included in the grantor's estate if the grantor dies within 3 years. C. The premium paid on the policy will be taxable to the grantor(s). D. The trustee will not do as instructed.

B The biggest risk is that an existing policy will be included in the grantor's estate. If they die within the 3 years the policy is included in the estate, but the proceeds are payable to the trust. It is a royal mess because the trust has the proceeds but the estate has the tax. If the trust purchases the policy, the policy will not be subject to a 3-year rule (Answer A). The premium paid on the policy will come from gifts made by the grantor(s). This is an unfunded trust not a funded trust (Answer C). Answer D could be true, but the trustees have a fiduciary responsibility.

Don and Bill own a business valued at $1,000,000. Some years ago, they signed a cross-purchase agreement using life insurance. Don is leaving to start a new business. Don wants to purchase his life insurance policy ($250,000) from Bill. Which of the following triggers a transfer for value problem? A. A transfer for value of the policy from Bill to Don B. A direct transfer for value of the policy from Bill to Don's wife to avoid the three-year estate inclusion rule C. A sale of the policy from Bill to the new business (corporation) that Don is starting D. A sale of the policy from Bill to Don Which of the following does not trigger a transfer for value problem? A. Transfer for value the policy to another family member B. Transfer for value the policy to a trustee of an irrevocable trust C. Transfer for value the policy to Don D. Transfer for value the policy to Don's wife

B The exceptions are a sale or transfer to the insured (Don) or a sale to a corporation in which the insured is a shareholder or officer. C The best way to avoid the transfer for value rule is to have the insured purchase the policy. Don is then free to make a gift of the policy to his wife, another family member, or a trust. Don must survive the 3-year contemplation of death rule to keep the life insurance face value out of his estate, but it is income tax-free. A transfer to Don's wife is a transfer for value.

Diane is a great photographer. She wants to donate one of her photos to the local art museum. Her cost for film developing, printing, mounting, and framing is $250. The art museum curator feels the photo is worth $3,000. If her AGI is $100,000, how much can she deduct for her gift to the museum? A. $0 - It is personal property. B. $250 C. $1,500 D. $3,000 E. 50% of AGI

B The tax deduction for a work of art created by the taxpayer is limited to basis. This is even true if the gift is to a museum. This is different than art purchased and then later donated to an art museum.

Tom and Alice Moore want to retire in 10 full years. They have $1,000,000 in investments earmarked for retirement. During their retirement years they want to draw out $150,000 at the end of each year. They feel their joint life expectancy will be 25 years after retirement. When the second person dies (after 25 years), they want to leave $500,000 family members. What return do they need to achieve if no other money is contributed to their investment account? A. 6.01% B. 6.64% C. 8.42% D. 10.11%

B This is an unequal cash flow question. For ten years they will let the investment grow. The number 10 is the number of times the 0 occurs consecutively. For 25 years, the $150,000 is distributed at the end of the year. CF->CLR->-1,000,000 Enter v->0 Enter v->10 Enter v->150,000 Enter v->25 Enter v->500,000 Enter v->IRR, CPT END MODE

Meg and Tom want to retire in 10 full years. They have $1,000,000 in investments earmarked for retirement. During retirement, they want to draw out $150,000 at the end of each year. They feel joint life expectancy will be 25 years after retirement. When the second person dies, they want to leave $1,000,000 to family members. What return do they need to achieve if no more money is added to their investment account? A. 6.01% B. 6.88% C. 6.91% D. 7.23%

B. 6.88% CF CLR WORK->-1,000,000 Enter v->0 Enter v->10 Enter v->150,000 Enter v->25 Enter v->1,000,000 Enter vv->IRR->CPT MUST BE IN END MODE FOR THIS QUESTION THE 10 IS THE NUMBER OF TIMES THE 0 OCCURS CONSECUTIVELY; 25 IS THE NUMBER OF TIMES 150,000 OCCURS CONSECUTIVELY

Jerry wants to save $90,000 for the down payment on an apartment building. He recently won a lottery in the amount of $50,000 that he can invest at an annual rate of 8% compounded semiannually. If this is the only amount he an invest for this goal, approximately how many years will it be before Jerry has accumulated $90,000? A. 14.99 B. 7.49 C. 15 D. 7.5

B. 7.49 Clr work->END->2 P/Y->-50,000 PV->90,000 FV->8 I/Y->CPT N = 14.99->14.99/2 (this is semiannual periods, so divide by 2 to get number of years) = 7.49

In regard to the Code of Ethics, certificants make a continuing commitment to learning and professional improvement. Which principle does this apply? A. Objectivity B. Competence C. Professionalism D. Diligence

B. Competence

If your grandfather died this year, which of the following items are includable in his gross estate? I. Paid $108,000 in gift tax this year II. A $2.0 million life insurance policy that you bought on his life 2 years ago III. An income tax refund that was received just before his death IV. $5.0 million in his revocable living trust V. A taxable gift of $12,330,000 A. I, II, IV, V B. I, III, IV C. I, IV D. III, IV E. IV

B. I, III, IV The $108,000 paid in gift tax is subject to the gross-up rule and included in the gross estate. The taxable gift is included in the tax base but not in the gross estate. The $108,000 is $12,330,000- 12,060,000 = $270,000 $270,000 x 40% = $108,000

Sally Jackson and Bob Evans have a small profitable business. They only employ a few part-time workers. This year they both expect to be in the 35% tax bracket. They don't have a retirement plan. What would you suggest? A. Defined benefit B. Profit sharing C. Solo 401(k) D. SEP IRA

C It never says S corporation or C corporation and it sounds like a partnership. The Solo 401(k) is is the best answer. Part-time (less than 1000 hours per year can be excluded then they can do a Solo 401(k). The SEP IRA would have to include the part-time workers. Also there is no age 50 catch up in the SEP IRA. It does not say but they could be over age 50 or close. Answers A and B are overkill for a two person retirement plan.

Which of the following are amounts received that will not be treated as income-first distributions under a MEC contract? A.Cash dividends B.Interest accrued on a policy loan (added to loan balance) C.Dividends retained by the insurer to purchase paid-up additions D.Dividends retained by the insurer as principal or interest on a policy loan

C "Income first" simply means taxes are owed on the income since it is not viewed as a return of principal. Answer C dividends are not treated as income-first distributions. The loan interest was not paid. It is added to the existing loan. It is like you made a loan against the loan. Answer C does not cause a MEC problem. They buy additional amounts of paid-up insurance each year. They are not distributed to the client but are in fact reinvested in the contract. NOTE: Interest paid on accumulated dividends does not affect a MEC answer. This is interest due on a loan from a MEC.

Jane purchased a deferred annuity for $150,000. The annuity provides monthly payments of $1,200. Jane's life expectancy is 25 years. How much of each payment will be included (taxable) in income? A. $0 B. $500 C. $700 D. $1,200

C $150,000/360,000 =41.66% excluded 1,200 x .4166 = 500

Pat, age sixteen, has earned income of $2,000 and interest income of $1,200. What is his standard deduction this year? A. $1,150 B. $1,200 C. $2,400 D. $3,200

C $2,000 + $400 = $2,400 earned is greater than $1,150 unearned. The $400 is the current additional standard deduction for a child with earned income this year.

Toby and Helen Booth want to buy a house. To get the 4% fixed mortgage for 30 years they have to put 20% down. They feel they can afford to pay a total of $2,500 per month (PITI). They have researched the property tax (probably $4,500 per year) and insurance (probably $1,500 per year). They have done their math and just want a confirmation from you on the maximum purchase price of the house they could buy? A. $418,922 B. $448,652 C. $523,652 D. $654,565

C 1st step: They have $30,000 per year less $6,000 expenses (taxes/insurance) or a net of $24,000 or $2,000 per month. 2nd step: CLR->12 P/Y->2,000 PMT->4 I/Y->30xP/Y N->CPT PV = 418,922 3rd step: $418,922 is 80%, they are putting 20% down.->418,922/0.8 = 523,652

Susan, age 45, is the wife of Harry, age 66. Harry just qualified for Social Security payments of $1,500 a month. Harry's pension plan is minimal. Harry cannot work any more due to physical limitations. Susan and Harry got married just 10 years ago. She wanted children and they have two in gradeschool. Can she qualify for Social Security benefit? A.No, she is under 62 B.No, Harry is not disabled C.Yes D.No, but the children can

C A spouse of a retired worker qualifies for Social Security benefits at any age if they have a child in care under the age of 16. Answer D only applies if Harry is deceased.

Jane Smithson, age 62, lost her job at the end of last year. She is currently collecting unemployment compensation. Since she is age 62, she decided to collect Social Security benefits early. She is getting benefits based on her ex-husband's Social Security. He is age 67. He has 5 years of alimony still to pay her. She received an alimony payment this year from the 2016 divorce settlement. She has a fair amount of invested assets that will produce sizable dividends and capital gains (distributed not re-invested). She wants to make a deductible IRA contribution. Can she? A. No, she has no compensation B. No, her AGI will be too high due to sizable dividends and capital gains C. Yes, she has compensation. D. Yes, she is getting unemployment compensation.

C Actually the compensation is alimony. Yes, alimony and separate maintenance payments received from a divorce finalized prior to 2019 are compensation. Unemployment compensation and Social Security payments are not compensation. She is not employed therefore there is no AGI phaseout.

What is the maximum amount of life insurance benefits that are permissable in a qualified retirement plan? A. Zero, life insurance is not a permissable investment in an IRA. B. $245,000 in a defined benefit plan C. 100 times expected monthly benefit amount in a defined benefit plan D. Up to 50% of the contribution can be used to buy whole life insurance based on age sensitive insurance

C Although A is correct, the question is asking about a qualified plan not an IRA. $245,000 is the benefit allowed, not the life insurance allowed. The 50% rule does apply, but 100 times expected monthly benefit amount will be substantially greater benefits. Answer D is partially correct. It is not based being on age sensitive. In addition it is substantially less than Answer C.

A CFP® professional, owned a dually registered firm with investment advisory and broker-dealer divisions. The CFP® professional likes to work as a registered representative because that is how he started in the business 30 years ago. Occasionally he refers those same clients to the advisory division of his firm. The CFP® professional is very casual about the referrals and does not disclose to the client that the divisions are related. The CFP® professional, of course, receives additional compensation through the firms receipt of investment advisory fees. The CFP® professional also does not mention there are other very good investment advisory firms within miles of his office. Should the CFP® professional, have any concerns about these actions? A. No, the firm is dually registered. B. No, he is just referring the client now making them use the advisory division. C. Yes, he violated Standard A.1 (place the client's interest ahead of his or her firm) and A.5 (present a summary of likely conflicts of interest). D. Yes, he should disclose that the parties are related.

C Although Answer D is true it never mentions specific standards. Yes, he should disclose the conflict of interest due to the related party status. He should also advise clients that they have the opportunity to seek investment advisory services from an unrelated firm.

Which of the following are dfferences between ETF's and mutual funds? A. ETF's are traded on the market on current day closing price. Mutual funds are bought and sold directly from the mutual fund company at NAV B. ETF's have no fees. Mutual funds can have two types of fees. C. ETF's are typically structured to shield investors from capital gain taxes. Mutual fund sales can trigger capital gains at sale. D. ETF's generally disclose holdings daily. Mutual funds only disclose holdings annually.

C Answer A is incorrect about ETF's. They trade throughout the day at the current market price. Answer B is incorrect about ETF's. ETF transactions may include brokeage commission. Answer D is incorrect. Mutual funds generally disclose holdings quarterly.

During the educational funding period (pre-college), which of the following techniques will work for someone with a $60,000 MAGI for their child? A. An UTMA funded with EE education bonds B. A Parent Loan to Undergraduate Student (PLUS) C. Yearly gifts by a grandparent to a 529 plan and yearly contributions to a Coverdell ESA by the parents D. A Pell Grant

C Answer A is wrong because EE education bonds cannot be owned by an UTMA. The child owns the UTMA, not an adult over age 24. PLUS and Pell Grants are available during the college years. The question is asking about the pre-college years. EE bonds are fine for an UTMA; using the bonds for education is not. EE bonds need to be owned by someone other than the student to qualify for the education tax break. If they are owned by a student in an UTMA and used for education, the tax break for education will be lost.

Charles Anthony is approaching retirement. Through the years, he has been spending more of his income (100,000+). He has been contributing to a 401(K) with a company match for a year. Charles has been divorced for years and his children are on their own. When he retires, he plans to do extensive traveling. His condo is paid for, but monthly expenses, taxes, and utilities are usually $1,500 per month. How much should he have in an emergency fund when he retires? A. Between the 401(K) and social security benefits, 3 months should be adequate B. 3 months because he is single with two sources of income C. 6 months because he is not financially well off D. At least one year of living expenses

C Answers A and B are basically the same and difficult to pick between. We do not know how much is in the 401(k). Social Security at NRA will probably be $3,000 per month. Travel expenses are unknown. The only two tested options are 3 and 6 months

Bobbie Langer is single. He owns a small company, a regular corporation. This year he earned $200,000. The company also paid him $25,000 in dividends and made a contribution of $50,000 to his account in the company profit sharing plan. He is saving about 40% of his earned income and has built up an investment account of $1.5 million. Last year, he made some short-term trades that netted him $50,000. How much did he pay in 2022 FICA taxes? A. $9,114.00 B. $10,534.05 C. $12,014.00 D. $15,300.00

C Bobbie owns a corporation. He is not self employed. He gets FICA wages. The calculation is 6.2% up to $147,000 and 1.45% unlimited. The .9% does not start until $200,000 for single and $250,000 for married individuals. There will be no questions above $200,000. 6.2% of $147,000 = $9,114.00 1.45% of 200,000 =2,900.00 Total = $12,014.00 The $50,000 contribution to the profit sharing plan is a company contribution. There is no deferral, then no FICA.

Lana Turner, age 68, caused an auto accident. She claimed she did not see the other car. She was found at fault. Under what parts of her auto policy will her medical claims be covered? A. None, she caused an at fault accident B. Under Part A (BI/PD) the BI (bodily injury) will cover her C. Under Part B (Med Pay) her medical claims would be covered D. Under Part C (uninsured motorist) her medical claims would be covered

C Both Part A and Part C cover liability claims. Please review these sections if you are unsure. Part C could indirectly reimburse her for her medical claims if she was not at fault. If she was not at fault and the at fault driver did not have insurance, then under Part C she could claim all her expenses not just medical claims.

Mr. Hart, married, has a son, Robert. Robert is now starting college. Mr. and Mrs. Hart are very happy that Robert has turned his life around. Robert was convicted of a felony for distributing a controlled substance. What can they do to save taxes in regards to education funding if their AGI is $100,000? A. Claim an American Opportunity Credit B. Make a deductible charitable gift of tuition to the college C. Claim a Lifetime Learning Credit D. Make a tax-free gift by paying Robert's tuition

C Both the American Opportunity Credit and Lifetime Credit programs spell out certain exclusions. An eligible student can be excluded if convicted of a drug related felony (distributing a controlled substance) under the American Opportunity Credit only. Lifetime Learning does not have that restriction. Their AGI qualifies them for a Lifetime Learning Credit. Answer D is an answer, but Answer C gives them a tax write-off. No, you cannot get a charitable deduction for a tuition gift to a school; there are just no gift tax implications. Answer D does not generate a tax write-off.

Tom purchased a bond for $950 that has a coupon rate of 6%. The bond matures in 17 years and is callable in 5 years at $1,110. What is the YTC for this bond? A. 4.53% B. 4.67% C. 9.05% D. 9.70%

C CLR->2 P/Y->5xP/Y N->-950 PV->1,110 FV->30 PMT->CPT I/Y END MODE

Charles Lee has had volatile returns over the years. He is concerned about retirement and wants to have $1 million saved in 6 years. If he invests $500,000 now and adds $50,000 at the end of every other year, what kind of IRR does he have to achieve? A. 7.21% B. 8.15% C. 8.63% D. 17.99%

C CLR->CF->-500,000 Enter v->0 Enter vv->-50,000 Enter vv->0 Enter vv->-50,000 Enter vv->0 Enter vv->950,000 Enter v->IRR, CPT (950= 1M - 50k at end of year 6)

Tommy and Roseanne Burns are a young couple and are just getting started financially. He earns $58,000 per year and she earns $30,000 per year. He fully participates ($3,480) in a 401(k) plan at work (their AGI is $71,720). They are currently in a 12% income tax bracket but expect to be in a higher tax bracket when they retire. What else should they do to further their retirement planning? A. Roseanne should contribute to a $6,000 deductible IRA to get a tax credit. B. Tommy and Roseanne should each contribute to a $3,000 deductible IRA. C. Tommy and Roseanne should each contribute to a $3,000 Roth IRA. D. Tommy should contribute $3,700 to a non-deductible IRA and $2,300 to a deductible IRA.

C Doing a deductible IRA in a 12% tax bracket creates a tax deduction of only $720 (Answer A). The Roth distributions at retirement will be entirely tax-free and contributions can be removed tax-free at any time. The Roth seems to make more sense with their low tax brackets now and higher tax brackets at retirement. CFP Board will probably want you to use a Roth. The question is trying to compare apples-to-apples. Yes, the $3,000 to a Roth may have confused you. But all the answers use $6,000 in total. Yes, they could have contributed $6,000 each. This is the exam - confusion.

Which of the following are considered assets of the student for financial aid calculations? A. EE education bonds B. Coverdell ESAs accounts C. UTMA account D. 529 plan accounts

C EE education bonds must be owned by a parent or some one age 24 or older.

Bob died. Which is the following will be included in his gross estate? A. A life insurance policy on Bob owned by Bob's daughter. She is the beneficiary. It has a death benefit of $10 million. B. $6 million of taxable gifts Bob made during his lifetime C. A general power of appointment Bob had on his deceased mother's trust D. Assets gifted to his ex-wife who is a resident alien

C General powers are included in the gross estate. The life insurance policy is owned by his daughter. The taxable gifts are added to the taxable estate. Answer D may or may not be taxable gifts, but again, they are added to the taxable estate. They are not in the gross estate

Based on the following facts, what is the holding period return of the following investment? Purchased 1,000 shares at $30 on January 1st Paid a $1 dividend on December 1st of the first year. Paid a dividend on December 1st of the second year. It was 5% greater than the prior year. Sold 1,000 shares at $41 on December 31st of the second year A. 38.33% B. 41.67% C. 43.50% D. 45% E. 46.67%

C HPR = ($41.00 + $1.00 + $1.05) - $30 / $30= 43.50%

Tommy and Sally Long, both age 55, have realized they will never have enough to retire. Both realize they will work until they die. To maintain their lifestyle they need both of their salaries. They both have some group life insurance at work and decided to purchase individual policies to supplement the survivor should one of them die. If Tommy dies first how much of the insurance death benefit will be included in his gross estate? — Group life policy of $200,000 on Tommy with Sally the beneficiary — 30-year term policy insurance of $1 million on Tommy with Sally the beneficiary — 30-year level term policy insurance of $1 million on Sally with Tommy the beneficiary — Group life policy of $150,000 on Sally with Tommy the beneficiary — First to die universal life policy of $1 million on Tommy and Sally with the survivor the beneficiary. The cash value is $5,000. A. $1,200,000 B. $2,000,000 C. $2,200,000 D. $2,205,000

C Included is the group life on Tommy of $200,000 plus the 30-year term of $1 million on Tommy. Since he has an incident of ownership on the 1st to die, it is included. But only $1 million. Unless it says the policy is an option B increasing death benefit, it is an option A. The cash value is not included. When no ownership of a policy is mentioned, the insured is assumed to be the owner of the policy. NOTE: Group life is normally company paid. Tommy has an incident of ownership. He can change the beneficiary. It will be included in his estate.

Jane Morrow married Tom Tucker late in life. As part of their planning before marriage they signed pre-nuptial agreements to keep their assets separate. Jane wants to buy 100 acres of farm land. Tom feels it will be a good investment but only wants a 30% interest. This is okay with Jane. If they buy the 100 acres for $250,000, how much will be included in Tom's probate estate if he dies first and the 100 acres of farm land is valued at $300,000? A. $0 B. $75,000 C. $90,000 D. $125,000 E. $150,000

C Just his 30% interest based on the FMV of the property at the time of death is subject to probate. This purchase is not JTWROS. It is tenancy in common (30% interest). The question said they wanted to keep their assets separate. Answer A would be correct if the land was held JTWROS. JTWROS between two spouses is always 50%, but it avoids probate at the first tenant's death.

All of the following are requirements for real property to qualify for and retain the special use valuation (2032A) EXCEPT which of the following? A. The decedent must have been a U.S. citizen or resident. B. The real property in question must pass to a qualified heir. C. Either the decedent or a member of the decedent's family must have materially participated in the operation of the farm or business for at least ten years immediately prior to the decedent's death. D. A written agreement must be filed with the estate tax return and must be signed by everyone who has an interest in the real property.

C Prior to death, the five-out-of-eight-years rule applies; after death there is a ten year rule.

Jane Knowitall has an ultra clean FINRA record and a solid book of business. Jane went through a messy divorce. As part of the divorce she received the large overpriced home. Thinking she was smart she let it go in a non-recourse short sale last year. Does she have a problem with any regulators? A. No, a short sale is a legal method of letting go of a home. B. Yes, she should have reported it to the CFP Board. C. Yes, she should have updated her Form U-4 within 30 days. D. No, this is not a client issue, it is a personal issue.

C Questions 14K and 14M of the U-4 require a registered person to disclose the existence of any tax liens, unsatisfied judgments, bankruptcies, and compromises with creditors on their Form U-4. Timely is within 30 days. This non-recourse short sale is going to go on a credit report and FINRA can easily find it. There is no mention of Jane being a CFP® professional.

Chad and Martha Lewis have been hit by all kinds of financial difficulties. Although both are employed they need cash to straighten out their day-to-day affairs. Chad has been saving into a deductible IRA each year because he is not subject to phaseout. Martha has been saving through her company's 401(k) and depositing money into a Roth IRA. What is the best source from which to take money? A. His IRA B. Her 401(k) C. Her Roth D. She should stop deferring into her 401(k)

C Roth contributions, no matter when deposited, come out first tax-free. His IRA will be subject to income tax plus probably a 10% penalty. There is nothing to indicate the 401(k) has loan provisions. Answer D will not produce instant cash.

Mr. Ball and Mr. Desmond are both attorneys. Their business, BaDe, PC (professional corporation) does not have a retirement plan. They want to establish a qualified plan. They want a plan that no one will personally contribute to and that BaDe, PC would not have to contribute to each year. They currently have only two other employees (part-time at 400 hours/year) and plan to hire another part-time attorney. Which of the following plans should a CFP® professional recommend? A. No plan fits B. SEP C. Profit sharing plan D. Profit sharing 401(k) plan E. Defined benefit plan

C Since no ages or salaries were given, the defined benefit plan can be eliminated. You cannot do DB unless the question indicates excessive profits. The wording may be poor in that "no one will have to contribute" means they did not want to defer personal income. This eliminates the 401(k). The SEP was eliminated based on the wording: qualified plan and having part-time employees. The profit sharing plan gives the company the flexibility of not having a contribution each year. It fits the question. The ERISA 1,000-hour rule would eliminate the part-time employees.

Tom Turner owns TT, Inc. TT is a small business that Tom normally limits the year end profit to $15,000. As he reviews his purchases during the year he finds he spent $5,000 for computer, $10,000 for office equipment, and $10,000 for a new truck (he traded in his old truck). What will be his Section 179 deduction for the year? A. $5,000 B. $10,000 C. $15,000 D. $20,000 E. $25,000

C The Section 179 deduction is limited to his profit. It does not matter which piece of equipment he uses. That is not part of the question.

Tom is a participant in an HSA. He wants to pay for LTC premiums with distributions from an HSA. Is he allowed to do that? A. No, the HSA can only pay for LTC expenses, not premiums. B. No, the HSA can only pay for qualified medical expenses. C. Yes, the HSA can pay for qualified LTC premiums (age-based). D. Yes, the HSA can pay for all LTC premiums and expenses.

C The amount of qualified long-term care insurance premiums that constitute qualified expenses is allowed. However, the LTC premiums are still limited to age-based limitations (as adjusted annually). Answer B does not specifically answer the question. This is an HSA not an FSA. Premium payments are not allowed in FSAs but are allowed in HSAs. Answer D does not limit the premium payment to age-based limitations. HSAs have the same limitations as medical itemized deductions.

Which of the following is true about the special use valuation (2032A)? A. The maximum amount by which the gross estate can be reduced is based on a percentage of the exemption amount for the current year. B. It is available to any and all decedents as long as the property is located in the United States. C. It allows an executor to elect to value a farm for federal estate tax purposes based on actual current use, as opposed to the fair market value of the property if it was sold for development purposes. D. A qualifying heir can be a non-family member as long as the property remains in qualified use for at least ten years.

C The applicable base amount is $750,000. It is indexed for inflation. It is well over $1 million currently. The decedent must have been a U.S. citizen or resident. The qualified heir must be a lineal descendent.

Mrs. Smith, age 90, gifts to the following persons: $200,000 to her grandson (His parents died in an accident.) $200,000 to her grandnephew $200,000 to her sister's daughter $200,000 to her housekeeper, age 50 $200,000 to her friend, age 85 How much of her $12,060,000 GST tax exemption did she use? A. $0 B. $184,000 C. $368,000 D. $592,000 E. $736,000

C The grandnephew ($184,000) and the housekeeper ($184,000) are skip-persons, but you may have forgotten to subtract the two annual exclusions ($16,000 each). The sister's daughter is not a skip person. The sister's granddaughter is a skip person. The grandson is no longer a skip person when both of his parents die. There is no generational gap. The grandson is no longer a skip person when both of his parents die. There is no generational gap.

A man, age 55, has a fair amount of assets. He is concerned about long-term care beyond 100 days. Which statements are correct? I. Medicare may pay for more than 100 days of care after a 20-day deductible. II. If he purchases a LTC policy, the policy may pay if he qualifies (ADLs). III. Medicaid may pay if he uses up his assets to below the state threshold. IV. His major medical insurance may pay if he qualifies. A. All of the above B. I, II, III C. II, III D. II E. III

C The key word is "may" in Answers II and III.

Loretta started receiving substantially equal annual payments from her IRA at age 58. Loretta stopped receiving payments at age 61 (a total of three payments). What will be the amount of the recapture tax she will be subject to? A. None, she has attained age 59½. B. None, she ceased receiving payments. C. The recapture amount will be 10% of the total annual payments received before she attained age 59½ , and interest. D. The recapture amount will be 10% of the three payments, and interest. E. The recapture amount will be 10% of the value of the IRA, and interest.

C The penalty only applies to distributions that were made before age 59½.

Mr. Baker owns Baker Industries, Inc. which is an S corporation. The corporation pays the premium of Mr. Baker's personally owned disability policy. Which of the following are true? I.Baker Industries can deduct the premium II.Benefits payable to Mr. Baker under the policy will be tax-free A. I B. II C. Both I and II D. Neither I nor II

C The premium payment passes through to Mr. Baker as taxable income (conduit principal). He pays taxes on it; therefore, the benefits are tax-free. For companies with less than 20 employees the rules are different.

Harry started taking substantially equal payments from his IRA at age 55. For three years, he took the required amount. Then, in year four, due to a substantial hardship, he withdrew an amount greater than the normal substantial equal payment. He took the additional $50,000 as a hardship. What penalty did he have to pay on the additional $50,000? A. None, because he did a hardship withdrawal of $50,000 B. None, because he did a qualified loan of $50,000 C. 10% of $50,000 D. 10% of the withdrawals in years 1, 2, and 3 plus interest, and 10% of $50,000 in year 4 E. There was no penalty because he was 59½

C The question is only asking about the $50,000, not the withdrawals from years 1-3. Yes, the withdrawals from years 1-3 would be subject to the 10% penalty, plus interest, but that is not what the question asked. The question says he is 55, not 59½. If it does not say 59½, then the person is not 59½. Hardship withdrawals full under 401(k) plans, not IRAs. Here a hardship withdrawal is just a withdrawal from an IRA due to a hardship.

Joan Thomas sells her $200,000 term life insurance policy to Linda Bell for $1,000. Joan dies five years later. Linda paid $4,000 in premiums over the past five years. Will the insurance proceeds be taxable? A. No, a term life insurance policy does not trigger transfer for value. B. $199,000 will be income taxable to Linda. C. $195,000 will be income taxable to Linda. D. $200,000 will be included in Joan's estate.

C The sale of the policy triggered transfer for value. Linda didn't die. Joan died. Linda will not net $200,000. Linda will get the proceeds subject to income tax. When Joan dies, it will not be included in her estate (no 3-year rule) because it is a sale.

Mrs. Hanes, age 65, has just applied for Medicare coverage. If she elects Parts A, B, and D, what is the most significant gap in coverage would be other than the deductibles and 80% copay? A. She will not get critical care coverage. B. She will not get coverage for the first three pints of blood. C. She will have to get coverage if she takes a Viking River Cruise in Europe. D. She will have to get coverage for dental care, eye glasses, or hearing aids.

C There is limited coverage for services in Canada, Mexico, and the Caribbean. Answer D is also a limitation, but Answer C is more significant. Skilled nursing home care up to 100 days is also a limitation but is not an answer. An international emergency could be life threatening and would be uncovered. Answer D would probably not be life threatening. She will get critical (emergency) care under Part A. Three pints of blood would be a low cost item compared to Answers C or D. Subjective

Mrs. Kalish, age 82, gifted the following assets over the past three years. 3 years ago: She gifted a stock portfolio with a basis of $1 million and a FMV of $1.5 million currently worth $2 million 2 years ago: She placed $2 million in a 5 year GRAT with a gift tax value of $1.25 million, currently worth $2.4 million. 1 year ago: She gifted a whole life policy with a face value of $1 million and an interpolated terminal reserve plus unearned premium of $100,000. Mrs. Kalish passed away today. Which of the following is true? A. All the assets will be included in her gross estate at FMV. B. If her daughter sells the stock, she will have to pay tax on $500,000 at LTCG rates. C. $3.4 million of the assets will be included in Mrs. Kalish gross estate. D. The stock is not included in her gross estate because she lived 3 years.

C There is no 3-year rule with the stock. It is a taxable gift, but it is added to the taxable estate. It is not in the gross estate. The stock was a gift. The daughter's basis is $1 million and the gain is $1 million. The GRAT (5 year) and the life insurance (3-year rule) are included in the gross estate. The GRAT at FMV and the life insurance at face value are included.

A 50-year-old client has the following insurance situation: - No disability insurance - No LTC insurance - An auto policy with $100,000/$300,000/$50,000 BI/PD coverage when the umbrella required $250,000/$500,000/$100,000 - $1,000 comprehensive major medical deductible - $1,000,000 umbrella policy What should a CFP® professional recommend the client do first? NOTE: Think simple A. Buy long-term disability insurance B. Buy long-term care insurance C. Increase the auto policy liability limit D. Decrease the comprehensive major medical deductible

C This answer is the simplest and fastest recommendation. Increasing the auto liability will allow the umbrella to work properly. We do not know enough about the client to do A, B, or D. Is he/she working, retired, married, etc? What is his/her health? The client is 50 years old. Disability insurance is subject to both health and financial underwriting. It will take time to accomplish.

What is an advantage to the annuitant of a single life immediate fixed annuity? A. Annuitant receives a fixed payment. B. All taxation is deferred. C. No residual value remains in the gross estate of the annuitant. D. The annuitant can outlive the stream of income.

C This question is asking about an immediate annuity advantage. Answer C can be an advantage because it has no value at death (single life) in a taxable estate. Answer A is a disadvantage because the payment will be fixed for life, no inflation adjustment. Annuities are often purchased with after tax money, so all taxation is not deferred.

Mrs. Blanchard lives in New York City (Manhattan). She is in a 22% federal tax bracket and pays 8% NY state tax and 3% NY city tax. If she purchases Treasury bonds that pay 5%, what is her after-tax rate of return? A. 3.35% B. 3.38% C. 3.9% D. 4.5% E. 5.6%

C Treasuries are not subject to state or city taxes but are subject to federal tax. 5% (1-.22) = 3.9%

Bob and Sally Everheart, ages 34 and 33, have four children between 3 and 11. Bob earns $90,000 a year. The company he works for has only medical and limited life insurance benefits. Sally is a stay at home mom. They have done no planning. They have listed their priorities. What should a CFP® professional advise is the most important? A. Establish 529 plans for their children B. Try to save 10-15% for retirement C. Establish an emergency fund D. Buy term life insurance on him and her to cover living, education, and mortgage needs

C Very subjective exam type question. At this point in their lives retirement issues are least important. Answers A, C, D should come before Answer B. Usually emergency funds and guardianships come first, but it always depends on the scenario given. In the real world the answer would be a combination of Answers C and D first, but that is not an option. He already has some level of life insurance at work, so Answer D is at least partially addressed. This leaves Answer C as the biggest issue. With no mention of life insurance at work, Answer D would be correct.

Jensen

Calculates the excess return that a portfolio generates over its expected return. Also known as alpha Measures how much of the portfolio's rate of return is attributable to the manager's ability to deliver above-average returns, adjusted for market risk. The higher the ratio, the better the risk-adjusted returns Consistently positive excess return = positive alpha Consistently negative excess return = negative alpha Portfolio return - CAPM = Portfolio return - (Riskfree + Bx(Market return-Riskfree)

Schedule D

Capital Gains and Losses

Bonnie will need to make payments of $10,000 per year 6 years from now. She will have to make a series of 5 yearly payments at the beginning of the 6th year. She feels she can make a 10% after-tax return on a fund she sets aside now. How much does she have to put into the fund today to make future payments? A. $21,398 B. $23,538 C. $25,892 D. $29,747

C. $25,892 1st. How much to make the payment? BGN->-10,000 PMT->10 I/Y->5 N-> CPT PV = -41,699 2nd. She needs to make the payments at the beginning of the 6th year; so she only has 5 years to fund $41,699, not 6 years. 41,699 FV->10 I/Y->5 N->CPT PV = -25,892

Portfolio XYZ has the following characteristics: Market value beginning of year 1: $100,000 Capital withdrawals: end of year 1 $4,000 end of year 2 $5,000 end of year 3 $6,000 Market value end of year 3: $120,000 Based on the portfolio characteristics, the dollar weighted rate of return is which of the following? A. 1.57% B. 1.67% C. 10.92% D. 12%

C. 10.92% CF CLR WORK->-100,000 Enter v->4,000 Enter vv->5,000 Enter vv->126,000 Enter v->IRR->CPT

CLAT and CLUT

Charitable Lead Annuity Trust Charitable Lead Unitrust Charitable lead trust works by donating payments out of the trust to charity for a set amount of time. After that period, balance is paid out to non-charitable beneficiary Annuity - fixed amount paid annually, fixed amount is % of initial value of the trust's principal. Amount never varies Unitrust - Principal revalued annually, payments determined by % of value each year

CRAT

Charitable Remainder Annuity Trust Donor (grantor) contributes assets to an irrevocable trust that donates to one/more charities while also paying fixed income to one/more beneficiaries in the form of an annuity Annuity value calculated as fixed % of initial value of trust assets, must be no less than 5% but no more than 50% Irrevocable, so terms can't be altered, ownership of the assets belongs to trust and not the donor. Lasts until donor dies or for a set period no longer than 20 years, after which any remaining funds are donated to the charitable beneficiaries Although CRAT is a tax-exempt entity and the donor gets an up-front tax deduction for assets donated to the CRAT, the trust income distributed to noncharitable beneficiaries is taxable as ordinary income The grantor, by donating property in-kind to the CRAT, allows the tax-exempt sale of the property, which preserves its fair market value by avoiding paying capital gains taxes

CRUT

Charitable Remainder Unitrust Donor (grantor) contributes assets to an irrevocable trust that donates to one/more charities and pays a % of the value of the trust each year to noncharitable beneficiaries The payments generally must equal at least 5% and no more than 50% of the fair market value of the assets, valued annually

Kiddie Tax Calculation

Child's income - $1,150 SD --> Next $1,150 at 10% --> Remainder at parent's rate --> $115 + Remainder at parent's rate = Kiddie Tax

ISO

Corporate benefit that gives an employee the right to buy shares of company stock at a discounted price Profit on qualified ISOs is usually taxed at the capital gains rate

Alice is in a 32% tax bracket. She owns $10,000 of public purpose municipal bonds. They pay her $280 in interest semiannually. What pretax yield on corporate bonds is comparable to the yield on Alice's municipal bonds? A. 2.8% B. 4.18% C. 5.6% D. 8.23%

D 28 x 2 / 1,000 = 5.6% 5.6 / 1 - 0.32 = 8.23%

During the college years, which of the following techniques will work for a family with $190,000 MAGI to pay the tuition for the 1st year of college? A. A 529 withdrawal to pay tuition and a Lifetime Learning Credit using the tuition payment B. Coverdell ESA withdrawal to pay tuition and an American Opportunity Credit using the tuition payment C. A Pell Grant and a 529 withdrawal to pay tuition D. A PLUS and a Coverdell ESA withdrawal to pay tuition

D A 529 or a Coverdell ESA withdrawal cannot be used to pay tuition and then to take a credit for the tuition paid (Answers A and B). A Pell Grant is needs-based. This is a wealthy family. In Answer B, the American Opportunity Credit is phased out.

Which of the following investments should you recommend go into a 2503(c) or B trust (accumulating trust) that are not distributing income now? A. High yield bonds B. Convertible bonds C. Mortgage REITs D. Public purpose municipals

D Accumulating trusts have 37% plus 3.8% Medicare tax rates above $13,450. Answers A, B, and C all produce ordinary income. Appreciating assets, like growth stocks, are also a good recommendation, but are not among the answer choices given.

Mr. and Mrs. Iverson sold their home for a $400,000 gain. They excluded the gain under Section 121. They used the proceeds of the home sale to purchase another home. Now, a year and a half later, they sold the second home for a $50,000 gain. How much of the gain must they report? A. -0- B. $12,500 C. $37,500 D. $50,000 E. $450,000

D After keeping the second residence more than two years, the exclusion would become available again. It does not say they sold because of a job change, divorce, or unforeseen circumstance. You cannot assume any unforeseen circumstance. If it was due to a job change, etc. then you could prorate the exclusion, but none is shown. The first home used the 121 exclusion. It is asking about second home.

Money deducted from an employee's pay into an FSA is subject to which of the following? I. Withholding taxes II. No withholding taxes III. FICA IV. No FICA A. I, III B. I, IV C. II, III D. II, IV

D An FSA allows money to be deducted from an employee's paycheck pre-tax.

Gail Goodrich, single, owned a small corporation, GG, Inc. Due to business reversals, she had to close the business, and the stock became worthless. Which of the following is true if the loss is $120,000? A. If she meets the requirements of Section 1244, she can take an ordinary loss of $100,000 and a capital loss of $3,000. The remaining $17,000 is a carry-forward loss. B. She can take a $103,000 short-term loss. C. She can take a $53,000 long-term loss. D. None of the above

D Answer A is wrong because she is single, not married. Section 1244 allows for a $100,000 ordinary loss for married filing jointly and a $50,000 ordinary loss for single persons. The correct answer for single is $50,000 ordinary loss plus $3,000 capital loss and a $67,000 carry forward. It does not say in the question that she made a 1244 election.

Terry and Sandra Willis, ages 65 and 64, have not taken Social Security yet. Their only income comes from a rental apartment they own. The net income is $120,000. What is their status in regards to an IRA or Roth IRA? A. They can only contribute to a non-deductible IRA. B. Because they do not have a retirement plan they can contribute to a deductible IRA. C. They can contribute to a Roth IRA. D. They cannot contribute to any type of IRA or Roth IRA.

D Rental income (passive or active) is not earned income. Answer D is correct. They cannot do any type of IRA or Roth IRA.

Tammy is getting a divorce. As part of a property settlement, she is getting 50% ($250,000) of her husband's IRA ($500,000). Out of the settlement, she would like to buy a car ($20,000) with cash. What should she do if her AGI is less than $100,000 this year (single)? A. Take a distribution for $20,000 and roll over $230,000 into her own IRA. The $20,000 will be subject to ordinary income tax but no 10% penalty (QDRO exception). B. Roll the $250,000 into her own IRA. If she pledges $20,000 of the IRA to make the car loan, only the loan interest would not be deductible (consumer interest). C. Roll the $250,000 into a Roth IRA and then withdraw $20,000 to buy the car. Only the $20,000 would be subject to ordinary income tax. D. Do a trustee-to-trustee transfer of the whole $250,000 to her IRA and then withdraw $20,000 to buy the car. Only the $20,000 would be subject to ordinary income tax and a 10% penalty.

D Answer A is wrong because this is not qualified plan money. QDROs only apply to qualified plan distributions. Answer B is wrong because pledging the IRA to make a loan makes the IRA subject to income tax and a 10% penalty. Doing Answer C would make the whole $250,000 subject to income tax and the $20,000 subject to a 10% penalty. During the 60 days, she might find a cheaper car or find more liquid assets to reduce the taxable event. This is the only possible answer.

Jerry is 30 years old. He has insurance to cover all the following risks. Which risk is most important to address? A. Death before retirement B. His house burning down C. Getting in a car accident D. Becoming disabled before retirement

D Answer D carries a 20-25% probability. The other answers are all closer to a 5% probability (or less). Disability insurance is important. The average person files an auto claim about once every 18 years, but those claims are far less expensive than long term disability claims.

Mr. Rich is gifting $100,000 cash to a political organization (an American political party). How will the gift be handled? A. If he has an AGI of at least $166,667, he can deduct the full $100,000 under current COVID-19 legislation. B. He will have to declare a taxable gift of $84,000. C. If he has an AGI of at least $333,333, he can deduct the $100,000. D. Gifts to political organizations are exempt from gift tax.

D Answer D is true because Mr. Rich is gifting to an organization not a charity. There is no tax deduction with a gift to a political party. They are exempt because they are fully excludible. They are not subject to the $16,000 annual exclusion. This could be called a deductible gift or exempt gift.

Which statements regarding a health FSA are incorrect? A. It may receive contributions from an eligible employee (individual) B. Allowable contributions are not subject to FICA. C. Allowed contributions are not included in income. D. Reimbursements from the health FSA used for expenses are not taxed.

D Answer D is wrong because it does not specify "qualified" medical expenses. Picky, but A, B, and C are correct. The answer must say qualified medical expenses.

-Robert Zimmerman owns his own company (X). The company (X) has a profit sharing 401(k). He defers the maximum, and with the company match and usually some forfeitures, annual additions range between $20,000 - $25,000. -He has started another company (Y) with some friends, and they are considering a profit sharing 401(k) plan. He is considered a controlling shareholder of the new company (Y). Which of the following is true? A. He cannot be a participant in the new company's (Y) profit sharing 401(k) plan. B. Since he is fully participating in X's plan, he cannot participate in Y's plan (related employers). C. He is limited to the lesser of 25% of covered compensation or $60,000 for annual additions provided by both X and Y. D. He cannot defer any income into the new company's (Y's) 401(k) plan.

D He can be a participant of Y's plan for profit sharing contributions but not make any more deferrals. Answer C is incorrect because his annual additions limit (for both plans combined) is 100% of compensation or $61,000 (controlling shareholder). He can be a participant in Answer A because he is not maxed out at $61,000. He is currently at around $40,000. You do not know if he is over age 50. Could defer to both if they weren't related

You have been advising Sam Diggs service related business. He started his business years ago. Lately Sam has been having trouble retaining employees. Wages are an issue but so are employee benefits. He wants to start a retirement program. He wants the employees to also contribute. Sam's business is an S corp with about 7-8 employees during the year. Sam's FICA income is $4,000 per month. He hates FICA taxes. What do you suggest? A. Install a SARSEP B. Install a profit sharing with a match up to 6% C. Install a profit sharing with a match up to 8% and a 5% profit sharing contribution D. Install a simple IRA

D He cannot do Answer A since 1996. There is an administrative expense with profit sharing plans. He is not going to benefit very much from Answers B and C. Without knowing more we would go with Answer D because is the cheapest and easiest. Economically we do not know what he can afford or what the employees make. He may want to offer other benefits. This is low cost and simple.

Toby retires at age 65 taking Social Security early (NRA 66). He is getting $2,500 a month. He is also taking distributions from his retirement plan of $4,000 a month. He gets $2,000 a month from municipal bond funds. If he is single, how much of his Social Security is taxable? A. None, he has no earned income B. $1 for every $2 over the threshold due to early retirement C. 50% D. 85%

D His MAGI is $87,000. 85% is taxable. Answer B would only apply if he had earned income. There is no earned income.

A client wants to buy a mutual fund that will perform nearly opposite the market. Which fund should he/she select? A. Mutual Fund A: alpha +2, beta 1.5, R2 (squared) 15% B. Mutual Fund B: alpha +1.9, beta 1, R2 (squared) 100% C. Mutual Fund C: alpha +1, beta .1, R2 (squared) 40% D. Mutual Fund D: alpha -1, beta -1.3, R2 (squared) 1.5%

D If a mutual fund has a beta of -1, this means that when the market moves up, this mutual fund moves down at the same rate (a counter-cyclical mutual fund such as gold fund would fall into this category). R2 affects whether you use alpha, Treynor, or Sharpe. R2 above 60% or below 60%. The R2 does affect the answer. In Answer D the R2 is nearly zero. To use Alpha, R2 must be above 60% on a diversified fund. This is a beta question based on the fun moving opposite the market

A client has a son and a daughter in college. They both had $3,000 of college expenses this year. How much American Opportunity Credit could the client claim? A. $2,500 because that is the maximum AOTC. B. $5,000 because $2,500 will be available for each child C. $6,000 based on their total expenses D. $4,500 because they did not have the maximum expenses allowed in figuring the credit

D The AOC is available for both children, unlike the Lifetime Learning Credit. It is calculated at 100% for the first $2,000 of expenses and then 25% of the next $2,000 of expenses.

Mark Spout created an irrevocable trust for the benefit of his dependent children. Mark named the local bank as trustee of the trust and authorized it to invest in stocks, bonds, and negotiable certificates of deposit. Included in the investment authority is the right to use trust income to purchase insurance on Mark's life. All funds are currently invested in high-yield bonds paying 7% semiannual interest on a par value of $100,000. Twenty-five percent of the bond investment income is being used to pay the premium on a policy on Mark's life. Which taxpayer must pay tax on the income of the trust and why? A. The bank because of its broad authority as trustee B. The children because the income is paid by them C. The trust because it is irrevocable with no benefits to grantor D. Mark because of the grantor rules

D If any portion of the trust income is, or may be, used to purchase insurance on the life of the grantor or grantor's spouse, then the trust is a grantor trust. The trust is/was also used to benefit his dependent children (support). 25% is to purchase insurance and the remainder is used for support. It is a tainted trust so all the income is taxed to him.

Mr. Thomas is age 72. He is married (second wife age 54) and has two sons (ages 40 and 36) by his prior marriage. He wants to take his RMD as slowly as possible. Which of the following is his best option? A. Take distributions, using the new uniform life expectancy table B. Take distributions with his son (age 36), using the joint and last survivor table C. Take distributions with his son (age 40), using the joint and last survivor table D. Take distributions with his wife, using the joint and last survivor table

D In 2001, they made the automatic payout for RMDs based on a 10 year spread. The only exception is a spouse. His wife is more than 10 years younger. The joint and last survivor table would be more favorable than the uniform life expectancy table using the more than 10 year spread chart.

Alex, age eight, has an UTMA account set up for him by his grandfather (24% marginal tax bracket). This year, the account generated $1,000 of interest and $2,600 of short-term capital gains. Alex's father (35% marginal tax bracket) is the custodian of the UTMA. What is Alex's tax liability this year? A. $115 B. $240 C. $427 D. $570 E. $1260

D Income $3,600($1,000 interest + $2,600 STCG) - standard 1,150 ->next 1,150 at 10%(his tax rate;$115)->remainder at 35% = 115 + (1,300 x .35) = 115 + 455 = 570

What investment is most suitable for a client seeking growth and tax efficiency? A. Limited partnership B. Municipal bond C. Government bond D. S&P Index fund

D Index funds seek to duplicate the S&P 500 Index. Turnover of stock is minimal; therefore, they are tax-efficient.

Beth Anderson is only in a 12% tax bracket. She has been buying junk bonds at a discount. She recently bought 100 bonds for $95,000 with a 6% coupon that will mature in 20 years. The bonds are callable in 10 years with a 10% premium. What is her yield-to-call (YTC)? A. 6.30% B. 6.77% C. 7.02% D. 7.41%

D It is callable at $1,100 ($1,000 plus 10%) per bond. The 10% is based on a par of $1,000 not $950. CLR->2 P/Y->-950 PV->1,100 FV->10xP/Y N->30 PMT->CPT I/Y

Which of the following is a deduction from AGI? A. Business loss B. Capital loss C. Alimony paid D. Standard deduction E. IRA distribution

D Itemized deductions or the standard deduction are deductions from AGI.

Jane Thompson, age 40, is divorced. Alex Thompson, her ex-husband, is remarried to Lola, age 25. Alex has been a bit tardy on making alimony payments since he married Lola. Jane wants her financial planner to meet with Alex. Jane is willing to pay her CFP® professional for their services and Alex is willing to meet. What should the CFP® professional do? A. Tell Jane there is a conflict of interest B. Make sure Alex learns nothing about Jane's financial affairs in the meeting. C. The CFP® professional think should about the potential conflict of interests further. D. The CFP® professional should refer Alex to another financial planner

D Jane and Alex want help. Answer D is a helpful answer that fits more with the CFP® professionals ethical responsibilities. Yes, there could be a conflict of interest due to Alex's late payments harming Jane, but that does not solve the problem. The answer needs to provide a solution.

John Jefferson has a net worth of $12 million. John's wife died 5 years ago. On her death bed, his wife made John promise that no matter what he would cater to the needs and wants of John, Jr. Junior never made it past his sophomore year. He flunked out. Junior cannot keep a job and has decided at age 30 to go back to college to complete his degree. The school has agreed to his re-enrollment. What should a CFP® professional recommend John Jefferson do to meet Junior's education costs? A.Establish of Coverdell ESA B.Establish a new 529 plan C.Set up a 2503(b) trust D.Pay the college cost including tuition directly E.Nothing, it is too late at age 30 for any education planning

D John Jefferson can still pay the full tuition (exempt gift) and gift $16,000 for other college related expenses. John made a promise. It is too late for Answers A and B. A 2503(b) only provides income not corpus. Answer D is a now answer (no time for A, B, or C).

A corporation purchased a whole life insurance policy for the life of a key employee. The corporation was both the owner and beneficiary under the contract. The corporation did not deduct the premium but instead listed the premiums paid as an asset on its financial statement. The corporation borrowed from the insurance company and various banks using the policy as collateral for the loans. Today, the corporation surrendered the policy for a gain. How will the gain be treated for income tax purposes? A. As a long-term gain because a termination of a right with respect to property is a capital asset of the taxpayer B. As a long-term gain because life insurance contracts are considered capital assets C. As a long-term gain because the cash value growth was not treated as ordinary income but it is treated as a capital asset D. As ordinary income E. As a return of basis, a nontaxable event

D Life insurance surrender gain above basis is always taxed as ordinary income.

Loise recently purchased a zero-coupon bond for $475. It will mature in 10 years, at which time it will be worth $1,000. What is the yield to maturity? A. 7.26% B. 7.13% C. 7.72% D. 7.58%

D NOTE: Zeros are calculated using semiannual periods (in end mode). CLR->-475 PV->1,000 FV->10xP/Y N->CPT I/Y

Harry Potter has a four year time horizon. Which of the following investments would you advise him to buy? A. Growth mutual funds B. REIT C. Balanced fund D. Immunized bond portfolio E. Gold fund

D Remember the definition of immunization. The duration of the portfolio is made equal to a pre-selected time horizon.

Mr. Segura is retired and living on the income from his investments. He recently placed 20% of his portfolio in a RELP. The RELP has an excellent manager and has return no less than 10% per year over the past 20 years. What is the biggest risk Mr. Segura faces with this alternative investment? A. The RELP is too aggressive for a retiree. B. He placed too much of his portfolio in the RELP. C. It may not generate enough income. D. He may be challenged to sell it if he needs to

D The Board most likely wants the liquidity issues with alternative investments to be the biggest risk. Many retirees invest in real estate because of the income it pays. 20% in a risky asset is still fine as well; many retirees live on the income of rental properties that consist of much more than 20% of their total holdings.

Which of the following is not an exception to early distributions (before age 59½) from an IRA? A. $10,000 for the purchase of a second home (first home was sold in 2010) B. Payment of a medical bill ($5,000) C. A single life annuity payment starting at age 40. D. Distribution in accordance with a QDRO

D The QDRO exclusion is for qualified plans only. The other answers are difficult, but you should know QDROs are not associated with IRAs. Answer A qualifies as a first time home since it has been more than two years since they have owned a home. A single life annuity will be 72(t). Answer B may or may not be an exception, but Answer D definitely is not.

Chris Towns is self-employed. His business has 10 employees. He is successful enough to fund various employee benefits for him and his employees. Which among the following existing benefits would not be added to his gross income or subtracted as a deduction on the 1040 to determine his AGI? A. Group life insurance benefit of $200,000 for him B. SEP contribution for him C. Net profit from his business of $250,000 D. A $6,000 IRA contribution by his wife E. Group health contribution for him and his wife of $7,200

D The group life is in excess of $50,000. The contribution excess is taxable compensation. The SEP deduction and self-employed health contribution are deductions for AGI. The net profit is taxable income. She is above the spousal phaseout for the IRA deduction. Yes, Answer C, net profit $250,000 (given) triggers the IRA phaseout. It is part of the data on the client. The group health premium was deductible as an adjustment to AGI. On the exam the whole question and the answer could contain relevant facts to answer the question. Expect the unexpected.

A home has a FMV of $450,000. The land is worth $100,000. The home is partially destroyed by fire. Damages are $100,000. The home is insured for $250,000 with replacement cost coverage. How much will the homeowner get (ignore the deductible)? A. -0- B. $55,556 C. $71,143 D. $89,286 E. $100,000

D The land is not insured. The replacement value of the home is $350,000. ($450,000 FMV less $100,000 land) Homeowners always uses 80% of replacement value unless the exam uses a different percentage. $250,000 divided by 80% of $350,000 ($280,000) multiplied by $100,000 equals $89,286. The ACV answer depends on the amount of depreciation. Without depreciation you cannot calculate ACV.

Sam was terminally ill. He sold his $250,000 life policy to a qualified viatical settlement company for $175,000 a year ago. In the past few days, he found a medicine that will prolong his life for 5-10 years. How is the $175,000 affected? A. He will have taxable income of $175,000. B. He will have to return the $175,000. C. The viatical settlement company is in trouble and could loose a lot of money. D. He will keep the $175,000, and it is tax-free.

D The viatical company's return will suffer if Sam lives too long, Answer C is true, but does not answer the question. People living longer than expected greatly affects the profitability of viatical settlement companies. Sam is not affected. He was terminally ill. Answer D answers the question.

What is the limit on total medical expenses that can be covered under a medical FSA? A. No limit B. $2,850 per person per year C. $5,000 per year D. $2,850 per year

D There is a cap on both the health FSA ($2,850) and the dependent care ($5,000). This cap applies to both contributions and reimbursements. The only way you could spend more than $2,850 is if the plan had a grace period or rollover feature, but that is not an answer choice.

Tony Roberts has been referred to you. On the initial phone call to set the appointment, he wondered how you could help him. You explained how you work with clients. He seemed to accept the explanation. At the initial meeting you supplied him with a list of information that he needed to gather. Now, at the data gathering meeting, he is supplying you with only limited data. Finally he explains he really only wants a specific portfolio recommendation. As a CFP® professional, which of the following should be done next? A. Terminate the relationship B. Limit the scope of the relationship C. Proceed as he wishes D. Redefine the scope of the engagement

D This is Practice Standard 1, but he changed the reason for his planning. Yes, Answers B and C are good. There is no reason to terminate the relationship (Answer A). You would go with Answer B when the client wants you to do something you cannot do, like wanting advice on a life insurance policy and refusing to give you their financial information. Answer D is more designed for situations like this where the client is asking for help you could deliver.

What is the current yield if a $1,000 bond with a 7% coupon is now selling for $945? A. 6.62% B. 7% C. 7.21% D. 7.41%

D This is a discount bond. Its current yield will be more than its coupon rate. CY = 70 / 945 = 7.41%

Which of the following is currently excluded from income? A. Interest income B. Unemployment income C. IRA distribution D. Alimony paid from a divorce finalized in 2019

D Unemployment income with no limit would not be excluded. Alimony received from divorces finalized after 2018 will not be included in taxable income

Terry starts a business as an S corporation with cash (capital of $1,000). He also personally lends the corporation $50,000. The business is now in need of additional capital. If the corporation applies for a commercial loan and he personally endorses it, will the commercial loan increase his basis? A. Yes, but only by $51,000 B. Yes, by the amount of the commercial loan C. Yes, by the amount of the commercial loan because he personally endorsed the loan D. No, because it is not a direct loan

D When an S corporation incurs a debt, no shareholder has any personal liability for the debt. Accordingly, no S corporation debt is included in a shareholder's stock basis, even if the shareholder has personally guaranteed the debt. If he personally took out the loan and then he lent the money to the S corporation, his basis would increase (direct loan).

Jack has an expensive wood framed house in the Blue Ridge mountains. With all the fires out west, what do you recommend he do? A. Sell the house B. Obtain the maximum homeowners insurance C. Increase the homeowners part A to the maximum D. Call and work with his property and casualty agent to obtain the best coverage available

D Yes, Answer C is a good answer, but Only a property and casualty agent can make the change. It could involve a change of carriers. Answer A is too extreme for Blue Ridge mountain properties. West properties are debatable on what to do.

T/F With an ILIT, death benefit passes down tax-free.

True

Rabbi Trust

Trust created to support the non-qualified benefit obligations of employers to their employees Creates security for employees because the assets within the trust are outside the control of employers; they are typically set up to be irrevocable

Which of the following plans is not subject to Federal income tax, FICA, nor FUTA on employee deferrals? A. Profit sharing 401(k) B. SIMPLE C. SARSEP D. 403(b) E. Section 125

E A 125 is a flexible spending account (FSA). All the other plans require FICA and FUTA on employee deferrals.

Which of the following is an example of a hazard? A. Wagering $100 on the outcome of Saturday's game B. Bringing a new product to market C. Purchasing a house D. The cause of a loss E. Building a house near a river

E Answers A and B are speculative risks. Answer C is a property risk. Answer D is a peril.

John is considering adding to his coin and stamp collection. Which of the following is true? A. This is an efficient market. B. Stamps and coins are more marketable than art and antiques. C. Stamps and coins have a small bid-ask margin. D. John would not be subject to the same elements of risk attributable to the stock market. E. The return on physical assets is normally non-correlated with returns on financial assets

E Inflation, while bearish to stocks and bonds, may be beneficial for collectibles. Answer D is a good answer, but Answer E is the best answer.

A put option with a strike price of $125 is selling for $4 when the market price of the underlying stock is $115. What is the intrinsic value of the put? A. 0 B. -6 C. 6 D. -10 E. 10

E It is $10 in the money. The fact that the premium is $4 is irrelevant to the intrinsic value.

A company has 18 full-time employees participating in their group health plan, and 4 full-time employees who are not participating in the plan. Joe, a participating employee with family coverage, just divorced Sara. How long will COBRA cover Sara and Debbie (his 12-year-old daughter)? A. Sara and Debbie gets 18 months. B. Sara and Debbie gets 36 months. C. Sara gets 36 months and Debbie gets 18 months. D. Sara gets 18 months and Debbie is still covered under the plan. E. Sara gets 36 months and Debbie is still covered under the plan.

E Sara will get 36 months (due to divorce). Debbie is still Joe's daughter and will continue to be covered by his group health plan (family plan). The plan is subject to COBRA (22 employees).

Which of the following would cause someone to recognize taxable income on their own tax return? I. A scholarship received by a student that was spent on a campus dormitory. II. A Pell Grant received by a student that was used to pay for an off campus apartment. III. A kiddie tax of $300 generated by an UTMA. IV. A student loan forgiven under an income driven repayment plan. V. A student loan forgiven under PSLF. A. All of the above B. III, IV, and V C. II, IV, and V D. I, II, IV, and V E. I, II, and IV

E Scholarships and grants are taxable income if not used for tuition, fees, or required supplies. A kiddie tax bill would cause the child's parents to pay taxes on their return, and that is not what the question asks. Under loan forgiveness, service related programs like PSLF are tax-free, but income-driven loan forgiveness methods produce taxable income when forgiven.

Holly, the daughter of Mr. and Mrs. Golightly, is going to college. She plans to get her Masters at a state university. Unfortunately, due to economic conditions, her parents never set up a 529 plan. She can qualify for some state scholarships. Her parents, both professionals, earn well over $100,000 each, but spend almost all they make. Which of the following may generate federal income tax credits for undergraduate as well as graduate education? A. American Opportunity Credit B. Lifetime Learning Credit C. Coverdell (ESA) D. PLUS E. None of the above

E The American Opportunity Credit may possibly work during the graduate years, but they are still above the $180,000 phaseout. The Lifetime Learning is subject to a phaseout at $160,000 - $180,000. The Coverdell and PLUS do not generate federal income tax credits.

What type of option offers the potential to make the highest profit for your client? A. Writing a covered put B. Writing a covered call C. Writing a naked put D. Writing a naked call E. Buying a call

E The upside of a call is unlimited. Answer A is not a bad answer because the stock's upside is unlimited, but to calculate the net profit of these positions (the short put and long stock), we must reduce the amount of gain by the original cost of the stock less the put premium income (see example below). For Answer E, the only offsetting cost is the call's premium. The call premium is almost certainly lower than the cost of the stock (assuming the stock and call were purchased at the same time). Answer E is the best answer because of the unlimited profit potential.

Mr. Smith's 401(k) plan had no beneficiary because at the time it was set up he was divorced. So, he named his estate the beneficiary. About five years later he married Pamela. He is now retired and has been taking distributions for three years. If he dies, what can Pamela do? A. Take distributions at least as rapidly as the schedule he had in effect B. Roll over the 401(k) remainder into her own IRA C. Take distributions beginning over her life expectancy by December 31st of this year D. Distribute the account to herself within 5 years of his death E. The 401(k) will be subject to the probate process, so she will need to petition the probate court for the account.

E There is no named beneficiary. The proceeds will be paid to the executor of his estate. They will be subject to the probate process. It never says anything about his will other than he named his estate the beneficiary. Remember this is a profit sharing plan, not a pension plan. It is not subject to QJSA. Whether he was required to name her as a beneficiary or not is immaterial. You must answer the question as written. All she can do is petition the probate court. The court will decide.

GRAT

Grantor Retained Annuity Trust Grantor transfers ownership of asset from his name to the trust Grantor retains ability to receive fixed payments on a regular basis based on a percentage of the fair market value of the asset at the time it was transferred to the trust

GRIT

Grantor Retained Income Trust Trust creator (grantor) transfers ownership of an asset from his or her individual name to ownership by the trust Asset is basically removed from the grantor's taxable estate. Grantor loses the ability to sell or transfer the asset, but retains the right to receive payments of income that the asset generates for a set number of years

GRUT

Grantor Retainer Unitrust Grantor transfers ownership of asset from his name to the trust Grantor receives a fixed percentage of FMV of the trust assets FMV calculated on an annual basis Percentage is fixed, but the amount paid to grantor varies based on changing value of the asset

Adjusted Gross Income (AGI)

Gross income minus adjustments(reported on schedule 1) Adjustments: Alimony, early withdrawal penalties, HSA deductions, student loan interest deduction, retirement contributions, self-employment tax, medical insurance premiums, etc.

HEMS Provision

Health, Education, Maintenance, and Support. When assets are distributed to the beneficiaries of a trust with a HEMS provision, the money can only be used for specific needs tied to the beneficiaries' health, education or living expenses

New Uniform Life Expectancy Table vs. Joint and Last Survivor Table

New Uniform used for all situations except when beneficiary is spouse more than 10 years younger Joint and Last Survivor used when beneficiary is spouse who is more than 10 years younger

Do Roth IRAs have RMDs?

No

Are zero-coupon bonds issued at par?

No They are issued at a deep discount and repay the par value, at maturity

Is rental income earned income?

No, it is passive income

Beta is a measure of ____, or _____ risk of a security or portfolio compared to the market as a whole.

Volatility, systematic

Schedule C is used to report

Income or loss from a business you operated or a profession you practiced as a sole proprietor

Call Option Intrinsic Value

IV = MP - EP

Transfer for value

If policy is transferred for something of value (money, property, etc.), a portion of death benefit is subject to taxation as ordinary income Portion is equal to the death benefit minus the item(s) of value, as well as any premiums paid by the transferee at the time of the transfer

2503(b)

Irrevocable trust, requires distribution of income on an annual basis

Do you use Treynor for an undiversified portfolio?

No, treynor only uses systematic (non-diversifiable) risk, so it assumes a portfolio is diversified, not as applicable to non-diversified portfolios

S Corp Keys

Keogh 100% medical, dental, LTC insurance deductible for 2% owner Losses up to basis (excluding corporate debt) Less than 100 shareholders, one class of stock, domestic corporation, shareholders must be US citizens/residents

Target Benefit Pension Plan (deduction, favors, cash, contributions)

Lesser of 100% of salary or $61,000 Up to 25% employer deduction Favors older employees Fixed mandatory contributions Stable cash flows Employer wants to provide a specific benefit at retirement

What is most bond interest taxed as?

Ordinary income

What is most interest income taxed as?

Ordinary income

Top-Heavy 401(k)

Owners and key employees own more than 60% of the value of the plan assets Employer must contribute up to 3% of compensation for all non-key employees still employed on the last day of the plan year

Single-life payout Joint-life payout

Payments will stop when the annuitant dies Payments continue after death to the annuitant's spouse

Medicare Part A

People 65+ eligible for SS benefits, disabled beneficiaries receiving benefits for 2 years Covers hospital stays, nursing home up to 100 days, home health services, hospice

Peril vs. Hazard

Peril is the CAUSE of loss. Hazard is a condition that increases LIKELIHOOD of loss.

Risk Retention

Planned acceptance of losses by deductibles, deliberate non-insurance, and loss-sensitive plans where some risk is consciously retained rather than transferred

Financial Planning vs Financial Advice

Planning - advice that integrates multiple aspects to meet goals Advice - recommendations that would reasonably be considered to cause clients to take/refrain from actions concerning their assets

Dividend Growth Model

Price = Next Dividend / Return - Dividend Growth Rate

What can cause active participant status?

Profit Sharing Defined Benefit Money Purchase Target Benefit 401(k) SEP SIMPLE 403(b)

QDOT vs QTIP

QDOT - Qualified Domestic Trust - Allows taxpayers who survive a deceased spouse to take marital deduction on estate taxes, even if surviving spouse is not a US citizen QTIP - Qualified Terminable Interest Property Trust - Enables grantor to provide for surviving spouse and maintain control of how trust assets are distributed after surviving spouse's death, allows for marital deduction - Surviving spouse has limited control of assets

Tax-Equivalent Yield (explained, formula)

Return that a taxable bond would need to yield in order to equal the yield on a comparable tax-exempt bond, such as a municipal bond Tax-Free Muni Bond Yield / (1 - Tax Rate)

Treynor (Hint: reward, volatility)

Reward-to-volatility ratio Determine how much excess return was generated for each unit of risk taken on by a portfolio Look for high R2 (60+) or diversified portfolio Look for highest positive alpha, then highest Treynor number ONLY USES SYSTEMATIC RISK = assumes diversified portfolio DOES NOT CONSIDER UNSYSTEMATIC (DIVERSIFIABLE) RISK Portfolio return - Risk-free rate / Beta

Required Rate of Return formula

Risk free + (Market return - Risk free)Beta

Sole Proprietorship keys

Risk-free business Keogh plan 100% of medical, dental, and LTC insurance is deductible for owner Deduct half of SE tax Lack of continuity

LLC keys

Risky business Limited liability Losses up to basis

Risk Avoidance

Run company in a way that eliminates certain hazards/exposures that could result in an expensive lawsuit/financial loss

C Corp keys

Separate Tax Entity using flat 21% tax rate Dividend-received deduction at least 50-100% Can offer NQDC Continuity of life Sell stock to unlimited number of investors

ISO Tax Treatment

Shares must be held for more than 1 year form date of exercise *AND* 2 years from time of grant in order for it profits to be taxed as capital gains

1244 Stock

Tax treatment of restricted stock by the IRS Allows losses from the sale of shares of small, domestic corporations to be deducted as ordinary losses instead of capital losses up to maximum of $50k for single, $100k for joint

NSO Tax Treatment

Taxed as ordinary income

_____ assets typically pass through probate

Titled

Profit Sharing Plan (deduction, favors, contributions, 401k)

Up to 25% employer deduction Flexible employer contributions up to 25%(must be recurring and substantial) 401(k) provisions - deferral up to $20,500 Use when profit or stability varies year to year Use when employer wants qualified plan with an incentive feature to motivate employees to make the company profit Employees young, well-paid, have time to save


Conjuntos de estudio relacionados

Inverse Sine, Cosine and Tangent

View Set

Managerial Accounting Chapter 11

View Set

Social Problems Chapter 4: Crime and Social Control

View Set

Chapter 14 Real Estate Financing

View Set